Vascular Flashcards

(150 cards)

1
Q

A 48-year-old woman presents for a diagnostic angiogram and is found to have bilateral
hypoplastic vertebral arteries. An angiogram (Figure 1) and a CT scan (Figure 2) are shown.
Through which of the following arteries is the posterior circulation being supplied?
Answers:
A. Vertebral artery
B. Persistent hypoglossal artery
C. Anterior choroidal artery
D. Posterior communicating artery
E. Persistent trigemeninal artery

A

Persistent hypoglossal artery

Discussion:
The angiogram demonstrates a persistent hypoglossal artery arising from the cervical segment of
the right internal carotid artery. An enlarged hypoglossal canal is seen on CT imaging. In early
embryonic development, there are 4 types of fetal carotid-vertebrobasilar anastomoses, which
include the primitive trigeminal, hypoglossal, otic, and proatlantal intersegmental arteries. As the
embryo develops, the posterior communicating arteries develop, and the anastomotic arteries
begin to regress at approximately the 30–40th days of fetal development. When this regression
fails, the embryonic arteries persist. The persistent hypoglossal artery is the second most common
carotid-vertebrobasilar anastomosis, arising from the cervical ICA and entering the posterior fossa
via the hypoglossal canal, before finally anastamosing with the terminal segment of the vertebral
artery.
References:
Chapter 3. Osbourn AG. Diagnostic Cerebral Angiography. 2nd Ed. Lippincott, Williams, and
Wilkins, 1999; Ozawa et al. Bilateral persistent hypoglossal arteries: a case report and literature
review. PMID: 31016350

How well did you know this?
1
Not at all
2
3
4
5
Perfectly
2
Q

A patient undergoes a left pterional craniotomy for aneurysm clipping. The aneurysm clip occludes
the small vessel indicated by the arrow in the image shown. Which of the following areas will
appear infarcted on CT scan?
Answers:
A. Left hypothalamus
B. Right thalamus
C. Left cingulate gyrus
D. Left caudate
E. Left optic nerve

A

Left caudate

Discussion:
The arrow points to the recurrent artery of Heubner, which can be injured during anterior
communicating artery aneurysm surgery. This vessel usually arises from the A2 segment of the
ACA, immediately after the Anterior Communicating Artery. The caudate head, anterior portion of
the lentiform nucleus, and anterior limb of the internal capsule are supplied by the recurrent artery
of Heubner.
References:
Ghika JA, Bogousslavsky J, Regli F. Deep perforators from the carotid system. Template of the
vascular territories. Arch Neurol. 1990:47(10):1097-1100.; Loukas M, Louis RG Jr, Childs RS.
Anatomical examination of the recurrent artery of Heubner. Clin Anat. 2006 Jan; 19(1):25-31

How well did you know this?
1
Not at all
2
3
4
5
Perfectly
3
Q

In 95% of the population, the vertebral artery enters the foramen transversarium at which of the
following cervical levels?
Answers:
A. C7
B. T1
C. C5
D. C4
E. C6

A

C6

Discussion:
The vertebral Artery usually enters the foramen transversarium of C6, to form the V2 segment. The
V2 segment ends as the artery leaves the foramen transversarium of C2.
References:
Osborn AG. Diagnostic Neuroradiology: A text/Atlas. 1st ed. Philadelphia: Mosby-Elsevier 1994;
Satti SR, Cerniglia CA, Koenigsberg RA. Cervical vertebral artery variations: an anatomic study.
AJNR Am J Neuroradiol. 2007;28 (5): 976-80.

How well did you know this?
1
Not at all
2
3
4
5
Perfectly
4
Q

A 45-year-old man with a presumed left-sided glomus jugulare tumor (paraganglioma) undergoes
preoperative angiography and embolization before surgical resection. The lateral projection left
common carotid angiogram (Figure A) and dedicated (selective) external carotid artery angiogram
(Figure B) are shown. Which of the following arteries is indicated by the arrow in both images?
Answers:
A. Posterior auricular artery
B. Lingual artery
C. Persistent primitive trigeminal artery
D. Superficial temporal artery
E. Ascending pharyngeal artery

A

Ascending pharyngeal artery

Discussion:
The ascending pharyngeal artery can be seen arising from the proximal external carotid artery
close to the source of the occipital artery and coursing superiorly.
References:
Osborn AG, Slazman KL, Jhaveri MD, et al, eds. Diagnostic Imaging: Brain 3e. 3rd ed. Elsevier,
2015; Cortés-Franco S, Muñoz AL, Franco TC, Ruiz T. Anomalous ascending pharyngeal artery
arising from the internal carotid artery: report of three cases. (2013) Annals of vascular surgery. 27
(2): 240.e1-4

How well did you know this?
1
Not at all
2
3
4
5
Perfectly
5
Q

A 60-year-old man is evaluated for acute onset of hemiplegia, hemisensory loss, and ipsilateral
gaze preference with no visual field deficit. Which of the following is the most likely location of this
patient’s occlusion?
Answers:
A. anterior choroidal artery
B. PICA
C. middle cerebral artery
D. AICA
E. cavernous internal carotid artery

A

middle cerebral artery

Discussion:
MCA stroke can result in contralateral hemiplegia and hemisensory loss as well as ipsilateral gaze
preference. In this case, primary motor and sensory areas are involved. Anterior choroidal stroke
results in homonymous hemianopsia.
References:
Ropper A, Samuels M, Klein J, eds. Adams and Victor’s Principles of Neurology. McGraw-Hill
Education; Afifi A, Bergman R, eds. Functional Anatomy: Text and Atlas. 2nd ed. New York, NY:
McGraw-Hill Education; 2005:362

How well did you know this?
1
Not at all
2
3
4
5
Perfectly
6
Q

The lenticulostriate arteries enter which of the following structures immediately after originating
from the middle cerebral artery?
Answers:
A. Globus pallidus interna
B. Caudate nucleus
C. Sylvian fissure
D. Anterior perforated substance
E. Internal capsule

A

Anterior perforated substance

Discussion:
The lenticulostriate arteries enter the anterior perforated substance immediately after originating
from the middle cerebral artery. The basilar tip perforators enter the posterior perforated
substance.
References:
Rosner et al. Microsurgical anatomy of the anterior perforating arteries. Journal of Neurosurgery,
61(3), 468–485. doi:10.3171/jns.1984.61.3.0468; Lawton MT: Middle Cerebral Artery Aneurysms.
In: Conerly K (ed). Seven Aneurysms: Tenets and Techniques for Clipping. New York: Thieme.
2011

How well did you know this?
1
Not at all
2
3
4
5
Perfectly
7
Q

Which of the following sets represents the proximal to distal order of vessels as they arise from the
aortic arch?
Answers:
A. Brachiocephalic trunk, left common carotid artery, left subclavian artery
B. Brachiocephalic trunk, right mammillary artery, left common carotid artery left subclavian
artery
C. Right subclavian artery, right common carotid artery, brachiocephalic trunk
D. Brachiocephalic trunk, right vertebral artery, left common carotid artery, left vertebral artery
E. Brachiocephalic trunk, right vertebral artery, left common carotid artery, left subclavian
artery

A

Brachiocephalic trunk, left common carotid artery, left subclavian artery

Discussion:
The aortic arch can exhibit normal variations in branching pattern. The most common variant,
comprising 80.9% of arches, is the classic pattern, consisting of the brachiocephalic trunk, left
common carotid artery, left subclavian artery. This pattern is followed by the bovine variant, where
the left common carotid arises from the brachiocephalic trunk (13.6%), and the left vertebral artery
variant, where the left vertebral artery arises directly from the aorta rather than from the left
subclavian artery (2.8%).
References:
Popieluszko et al., J Vasc Surg 2018 PMID 28865978. Osborn AG. Diagnostic Neuroradiology: A
text/Atlas. 1st ed. Philadelphia: Mosby-Elsevier; 1994.

How well did you know this?
1
Not at all
2
3
4
5
Perfectly
8
Q

A 28-year-old woman with aqueductal stenosis is undergoing endoscopic third ventriculostomy.
Which of the following veins is indicated by the arrow in the intraoperative image shown?
Answers:
A. Septal Vein
B. Thalamostriate vein
C. Caudate vein
D. Venous Angle
E. Terminal Vein

A

Septal Vein

Discussion:
The image shows a view through the foramen of monro from the right lateral ventricle. The
columns of the fornix form the anterior and superior margins of the foramen. The anterior septal
vein passes posteriorly along the septum and crosses the column of the fornix.
References:
Anderson RCE, Walker ML. Neuro-endoscopy. In: Albright AL, Adelson PD and Pollack IF, eds.
Principles and Practice of Pediatric Neurosurgery. New York: Thieme; 2007: 133; Osborn AG.
Diagnostic Neuroradiology: A text/Atlas. 1st ed. Philadelphia: Mosby-Elsevier 1994

How well did you know this?
1
Not at all
2
3
4
5
Perfectly
9
Q

Which of the following is the main function of occludin and claudin?
Answers:
A. Axonal transport
B. Epithelial tight junction maintenance
C. Regulation of clotting cascade
D. Inflammatory mediation
E. Glycolysis

A

Epithelial tight junction maintenance

Discussion:
Occludin and claudin are tight junction proteins that maintain blood brain barrier integrity.
References:
Neurology in Clinical Practice, Volume 2. 5th ed. 2008: 1695; Tsukita and Furuse,Overcoming
barriers in the study of tight junction functions: from occludin to claudin, https://doi.org/10.1046
/j.1365-2443.1998.00212.x

How well did you know this?
1
Not at all
2
3
4
5
Perfectly
10
Q

Which of the following is the primary source of the venous blood carried from the inferior petrosal
sinus to the internal jugular vein?
Answers:
A. Cavernous sinus
B. Internal cerebral vein
C. Emissary veins
D. Basal vein of Rosenthal
E. Vein of Trolard

A

Cavernous sinus

Discussion:
The origin of the inferior petrosal sinus is the cavernous sinus. The cavernous sinus drains to the
superior and inferior petrosal sinuses. The superior and inferior petrosal sinuses do not receive
drainage from the deep venous system. In the non-pathologic setting, emissary veins play a
relatively minor role in the circulatory system of the brain.
References:
Nolte J. The Human Brain: An Introduction to Its Functional Anatomy. St. Louis: Mosby; 1999;
Osborn AG. Diagnostic Neuroradiology: A text/Atlas. 1st ed. Philadelphia: Mosby-Elsevier; 1994

How well did you know this?
1
Not at all
2
3
4
5
Perfectly
11
Q

From proximal to distal, which of the following is the most common order of the internal carotid
artery branches?
Answers:
A. posterior communicating, ophthalmic, anterior choroidal, anterior cerebral, middle cerebral
B. posterior communicating, anterior choroidal, ophthalmic, anterior cerebral, middle cerebral
C. ophthalmic, posterior communicating, anterior choroidal, anterior cerebral, middle cerebral
D. ophthalmic, anterior choroidal, posterior communicating, anterior cerebral, middle cerebral
E. anterior choroidal, ophthalmic, posterior communicating, anterior cerebral, middle cerebral

A

ophthalmic, posterior communicating, anterior choroidal, anterior cerebral, middle cerebral

Discussion:
The ophthalmic artery is the first branch of the ICA distal to the cavernous sinus. The posterior
communicating artery typically arises proximal to anterior choroidal artery.
References:
Carpenter MB, ed. Core text of neuroanatomy. 4th ed. Baltimore, MD: Lippincott, Williams &
Wilkins, 1991; Osborn AG. Diagnostic Neuroradiology: A text/Atlas. 1st ed. Philadelphia: MosbyElsevier 1994

How well did you know this?
1
Not at all
2
3
4
5
Perfectly
12
Q

The inferior hypophyseal artery arises from which of the following?
Answers:
A. Superior hypophyseal artery
B. Thyrocervical trunk
C. Clinoidal internal carotid
D. Meningohypophyseal trunk
E. Posterior cerebral artery

A

Meningohypophyseal trunk

Discussion:
The inferior hypophyseal artery arises from the meningohypophyseal trunk and supplies the
posterior pituitary. The meningohypophyseal trunk arises from the posterior genu of the cavernous
internal carotid.
References:
Yasargil. Microneurosurgery. Vol 1; Osborn AG. Diagnostic Neuroradiology: A text/Atlas. 1st ed.
Philadelphia: Mosby-Elsevier; 1994

How well did you know this?
1
Not at all
2
3
4
5
Perfectly
13
Q

Which of the following arises from the internal carotid artery below the level of the petrolingual
ligament and supplies blood to a large glomus jugulare tumor?
Answers:
A. Labyrinthine artery
B. Otic artery
C. Caroticotympanic artery
D. Anterior choroidal artery
E. Vidian artery

A

Caroticotympanic artery

Discussion:
The petrous portion of the internal carotid artery has been described to branch out into the
caroticotympanic branch, the pterygoid canal (Vidian) and the periosteal
branch. A glomus jugulare tumor may extend anteriorly to involve the carotid canal and
caroticotympanic artery territory.
References:
Osborn AG, ed. Diagnostic Cerebral Angiography. 2nd ed. Lippincott, Williams & Wilkins, 1999:71;
Andreo et al. https://doi.org/10.1159/000016745; Hesselink AJNR 2:289-297, July/ August 1981

How well did you know this?
1
Not at all
2
3
4
5
Perfectly
14
Q

The internal auditory artery most often arises from which of the following arteries?
Answers:
A. Superior cerebellar artery
B. Posterior choroidal artery
C. Posterior Inferior Cerebellar Artery
D. Anterior Inferior Cerebellar Artery
E. Middle Meningeal Artery

A

Anterior Inferior Cerebellar Artery

Discussion:
The internal auditory artery, or Labyrinthine artery, originates from the AICA, although some
studies have reported that as many as 17% arise from the Basilar Artery.
References:
Kazawa N, Togashi K, Ito J. The anatomical classification of AICA/PICA branching and
configurations in the cerebellopontine angle area on 3D-drive thin slice T2WI MRI. Clin Imaging.
2013 Sep-Oct;37(5):865-870. Epub 2013 Jun 14.; Rhoton A. Cranial Anatomy and Surgical
Approaches. Neurosurgery. 2003; 53(3): 545

How well did you know this?
1
Not at all
2
3
4
5
Perfectly
15
Q

Which artery of the following branches of the external carotid artery provides the blood supply to
the lower cranial nerves?
Answers:
A. Superior thyroid artery
B. Ascending pharyngeal artery
C. Internal maxillary artery
D. Occipital artery
E. Lingual artery

A

Ascending pharyngeal artery

Discussion:
Cranial nerves IX, X, XI, XII are supplied by distal branches of the neuromeningeal trunk arising
from the ascending pharyngeal artery.
References:
Ozanne A, Pereira V, Krings T, et al. Arterial vascularization of the cranial nerves. Neuroimaging
Clin N Am. 2008 May;18(2):431-439, xii.Krisht A, Barnett DW, Barrow DL, et al. The blood supply
of the intracavernous cranial nerves: an anatomic study. Neurosurgery. 1994 Feb;34(2):275-279

How well did you know this?
1
Not at all
2
3
4
5
Perfectly
16
Q

The persistent hypoglossal artery is a connection between the basilar artery and which of the
following arteries?
Answers:
A. Petrous internal carotid artery
B. External carotid artery
C. Cervical internal carotid artery
D. Anterior cerebral artery
E. Cavernous internal carotid artery

A

Cervical internal carotid artery

Discussion:
The persistent hypoglossal artery arises at the level of C1 to C3 as a robust branch from the
cervical internal carotid artery (ICA). The persistent trigeminal artery arises from the cavernous
ICA, while the persistent otic artery arises from the petrous ICA.
References:
Brismar. “Persistent Hypoglossal Artery, Diagnostic Criteria: Report of a Case.” Acta Radiologica
Diagnosis, 1976. PMID: 1274653; Fujita N, Shimada N, Takimoto H et-al. MR appearance of the
persistent hypoglossal artery. AJNR Am J Neuroradiol. 1995;16 (4): 990-2. AJNR Am J
Neuroradiol (abstract) - Pubmed citation

How well did you know this?
1
Not at all
2
3
4
5
Perfectly
17
Q

A 45-year-old woman is evaluated because of a debilitating pulsatile tinnitus and worsening
headaches. In the lateral right common carotid angiogram shown, the arrows indicate branches of
which of the following feeding arteries to this high-risk dural arteriovenous fistula?
Answers:
A. Meningohypophyseal trunk
B. Anterior choroidal artery
C. Occipital artery
D. Middle meningeal artery
E. Superior Hypophyseal Artery

A

Occipital artery

Discussion:
Transcranial branches of the occipital artery are seen supplying this transverse-sigmoid sinus dural
arteriovenous fistula
References:
Osborn AG, ed. Diagnostic Cerebral Angiography. 2nd ed. Philadelphia, PA: Lippincott, Williams &
Wilkins, 1999; Susan Standring. Gray’s Anatomy. ISBN: 9780702052309

How well did you know this?
1
Not at all
2
3
4
5
Perfectly
18
Q

Occlusion of which of the arteries shown in the frontotemporal operative exposure would result in
an infarct localized to the anterior limb of the internal capsule?
Answers:
A. Anterior choroidal artery
B. Posterior communicating artery
C. M1 branch of the middle cerebral artery
D. Recurrent artery of Heubner
E. Distal anterior cerebral artery

A

Recurrent artery of Heubner

Discussion:
The recurrent artery of Heubner (also known as the medial distal striate artery) arises just distal
(and less commonly proximal) to the anterior communicating artery. It supplies the head of the
caudate, anterior limb of the internal capsule, anterior putamen and globus pallidus, the septal
nuclei, and the inferior frontal lobe. It is termed recurrent due to its course from its origin
backwards towards the ipsilateral proximal A1. The artery is often at risk of injury during surgical
treatment of an anterior communicating artery aneurysm and may develop subsequent
vasospasm. Compromise of blood flow in the recurrent artery of Heubner classically results in
contralateral arm and face weakness, and occasionally dysarthria. Bilateral injury results in
akinetic mutism. Involvement of the distal anterior cerebral artery would result in infarctions in
paramedian cortices. Vascular compromise of the anterior choroidal artery primarily affects the
posterior limb of the internal capsule. Compromise of the M1 branch would result in hemispheric
infarction throughout the middle cerebral artery territory. Finally, compromise of the posterior
communicating artery may or may not result in infarction, depending on collateral flow through the
posterior circulation.
References:
Falougy et al., Biomed Res Int 2013 PMID 23936853. Loukas M, Louis RG, Jr., Childs RS.
Anatomical examination of the recurrent artery of Heubner. Clin Anat. Jan 2006;19(1):25-31

How well did you know this?
1
Not at all
2
3
4
5
Perfectly
19
Q

Which of the following is the most proximal segment of the posterior inferior cerebellar artery
(PICA) that may be sacrificed with little risk of neurological compromise?
Answers:
A. Telovelotonsillar
B. Lateral Medullary
C. Cortical branches
D. Anterior Medullary
E. Tonsillomedullary

A

Telovelotonsillar

Discussion:
The PICA can be be divided into five segments. The proximal most segments (anterior and lateral
medullary) segments contribute branches to the brainstem. The tonsillomedullary segment is a
transitional zone which may produce some perforating vessels. The distal most segments
(telovelotonsillar and cortical) do not supply blood to the brainstem. The first three segments of the
PICA are usually preserved, while the last 2 can be sacrificed without major neurological deficits.
References:
Lewis et al. Distal posterior inferior cerebellar artery aneurysms: clinical features and
management. Journal of Neurosurgery. 2002; Lister JR, Rhoton AL Jr, Matsushima T, et al.
Neurosurgery. 1982 Feb;10(2):170-199

How well did you know this?
1
Not at all
2
3
4
5
Perfectly
20
Q

The blood-brain barrier effectively prevents the passage of which of the following into the brain?
Answers:
A. Oxygen
B. Glucose
C. Carbon dioxide
D. Amino acid
E. Gadolinium

A

Gadolinium

Discussion:
Small lipophilic substances such as oxygen and carbon dioxide diffuse freely across plasma
membranes of the blood-brain barrier along their concentration gradient. Nutrients including
glucose and amino acids enter the brain via transporters. Gadolinium only enters the brain at sites
of blood-brain barrier disruption.
References:
Ballabh et al. The blood-brain barrier: an overview: structure, regulation, and clinical implications.
PMID: 15207256; Citow J, Macdonald R, Puffer R et al., Comprehensive Neurosurgery Board
Review, ed. 3rd Edition. Thieme; 2019. doi:10.1055/b-005-148973

How well did you know this?
1
Not at all
2
3
4
5
Perfectly
21
Q

The anterior choroidal artery supplies blood to which of the following structures?
Answers:
A. Arcuate nucleus
B. Optic tract
C. Pineal gland
D. Gyrus rectus
E. Medial longitudinal fasciculus

A

Optic tract

Discussion:
The anterior choroidal artery most often arises from the internal carotid artery distal to the posterior
communicating artery. It supplies the posterior limb of the internal capsule, the optic tract, the
lateral geniculate nucleus, the medial temporal lobe, and the globus pallidus pars interna (GPi).
Ischemia of its territory classically results in contralateral hemisensory loss, hemiparesis, and
hemianopia.
References:
Hupperts, R. M. M., et al. “Infarcts in the anterior choroidal artery territory Anatomical distribution,
clinical syndromes, presumed pathogenesis and early outcome.” Brain 117.4 (1994): 825-834;
Yasargil, Microneurosurgery Volume 1

How well did you know this?
1
Not at all
2
3
4
5
Perfectly
22
Q

A microvascular decompression is performed through a suboccipital craniotomy on a 65-year-old
man with right V3 trigeminal neuralgia. Which of the following venous structures is seen in the
middle of the operative field during the approach to the trigeminal nerve in the image shown?
Answers:
A. Superior Petrosal Vein
B. Superior Petrosal Sinus
C. Superior Vermian Vein
D. Internal Cerebral Vein
E. Vein of Rosenthal

A

Superior Petrosal Vein

Discussion:
The superior petrosal vein or “Dandy’s vein” is seen. Initially described by Walter E. Dandy in its
anatomical relation to the trigeminal nerve and cerebellum, the superior petrosal vein is known as
an important venous drainage system in the posterior cranial fossa and is an important landmark in
the retrosigmoid approach to the trigeminal nerve. The vein empties into the superior petrosal
sinus.
References:
Watanabe T, Igarashi T, Fukushima T, et al. Anatomical variation of superior petrosal vein and its
management during surgery for cerebellopontine angle meningiomas. Acta Neurochir (Wien). 2013
Oct;155(10):1871-8; Blue et al. World Neurosurg. 2020 Jun;138:e420-e425. doi:
10.1016/j.wneu.2020.02.142

How well did you know this?
1
Not at all
2
3
4
5
Perfectly
23
Q

A 37-year-old man undergoes surgery for a right posterior communicating artery aneurysm.
Postoperatively, he is fully awake and alert and is following commands, but there is a dense
hemiparesis involving the left face, arm, and leg, as well as a left visual field cut. Which of the
following blood vessels was most likely compromised during the surgery?
Answers:
A. Posterior Medial Choroidal
B. Recurrent Artery of Heubner
C. Anterior choroidal artery
D. Lenticulostriate arteries
E. Posterior Lateral Choroidal

A

Anterior choroidal artery

Discussion:
The Anterior Choroidal Artery supplies the posterior limb of the internal capsule, the optic
tract/radiations and globus pallidus internus. Damage to this artery results in contralateral
hemiplegia, contralateral hemianesthesia and homonimous hemianopsia.
References:
Spetzler RF, Kalani MYS, Nakaji P, eds. Neurovascular Surgery. Thieme:25, 512; Hupperts, R. M.
M., et al. “Infarcts in the anterior choroidal artery territory Anatomical distribution, clinical
syndromes, presumed pathogenesis and early outcome.” Brain 117.4 (1994): 825-834

How well did you know this?
1
Not at all
2
3
4
5
Perfectly
24
Q

Which of the following arteries is indicated by the arrows in the cadaveric dissection photograph
shown?
Answers:
A. Posterior cerebral artery
B. Posterior communicating artery
C. Basilar artery
D. Superior cerebellar artery
E. Anterior inferior cerebellar artery

A

Superior cerebellar artery

Discussion:
The superior cerebellar artery can be seen arising from the distal basilar artery just before the
posterior cerebral artery. The occulomotor nerve travels between the superior cerebellar and
posterior cerebral arteries.
References:
Rhoton AL, Apuzzo MLJ, eds. Cranial Anatomy and Surgical Approaches. 1st ed. Lippincott
Williams & Wilkins, 2003; Osborn AG, Slazman KL, Jhaveri MD, et al, eds. Diagnostic Imaging:
Brain 3e. 3rd ed. Elsevier, 2015

How well did you know this?
1
Not at all
2
3
4
5
Perfectly
25
The pituitary gland is supplied by branches of which of the following arteries? Answers: A. Anterior cerebral artery B. Recurrent artery of Huebner C. Anterior communicating artery D. Middle cerebral artery E. Internal carotid artery
Internal carotid artery ## Footnote Discussion: The pituitary gland is supplied largely by branches of the internal carotid artery (ICA), including the superior hypophyseal artery, the prechiasmal artery (branching off the ophthalmic artery), and branches of the cavernous ICA (the inferior hypophyseal artery, the capsular artery, and the inferolateral trunk). There is also supply from the infundibular artery branching off the posterior communicating artery. References: Gibo et al. “Arteries to the pituitary.” Nihon Rinsho, 1993. PMID: 8254920; Yasargil. Microneurosurgery. Vol 1
26
A patient undergoes clipping of an anterior communicating artery aneurysm. The patient awakens slowly from anesthesia and has a mild contralateral hemiparesis. Which of the following vessels indicated by the arrow in the photograph shown is the most likely cause of such an injury? Answers: A. middle cerebral artery B. ophthalmic artery C. anterior choroidal artery D. recurrent artery of heubner E. lenticulostriate artery
recurrent artery of heubner ## Footnote Discussion: The recurrent artery of heubner exits A2 and traverses posteriorly. This artery can be inadvertently injured during clipping of anterior communicating artery aneurysms causing contralateral weakness. References: Gomes F, Dujovny M, Umansky F, et al. Microsurgical anatomy of the recurrent artery of Heubner. J Neurosurg. 1984 Jan;60(1):130-9; Loukas M, Louis RG, Jr., Childs RS. Anatomical examination of the recurrent artery of Heubner. Clin Anat. Jan 2006;19(1):25-31
27
Perforators off of which of the following arteries provide the blood supply to the subthalamic nucleus? Answers: A. Calcarine artery B. Posterior choroidal artery C. Posterior cerebral artery D. Superior cerebellar artery E. Anterior choroidal artery
Posterior cerebral artery ## Footnote Discussion: The posterior choroidal arteries arise from the P2 segment of the posterior cerebral artery and are made up of a number of branches. One to 2 medially placed branches arise from the distal P1 or proximal P2 segment of the posterior cerebral artery. These supply the subthalamic nucleus and midbrain, the medial half of the medial geniculate nucleus, the posterior parts of the intralaminar nuclei of the thalamus, and the pulvinar nuclei. References: Schmahmann JD. Vascular syndromes of the thalamus. Stroke. 2003 Sep;34(9):2264-78. Epub 2003 Aug 21; Haines DE. Neuroanatomy. Lippincott Williams & Wilkins. (2008) ISBN:0781763282
28
A 50-year-old woman has mild left hemiparesis on awakening after undergoing clipping of an unruptured intracranial aneurysm. Postoperative CT scan of the head shows an infarct involving the head of the right caudate nucleus, the anterior limb of the right internal capsule, and the anterior right putamen. Which of the following arteries was most likely occluded? Answers: A. Frontopolar branch B. Anterior communicating artery C. Lateral lenticulostriate artery D. Anterior choroidal artery E. Recurrent artery of Heubner
Recurrent artery of Heubner ## Footnote Discussion: The caudate head, anterior portion of the lentiform nucleus, and anterior limb of the internal capsule are supplied by the recurrent artery of Heubner, which can be injured during anterior communicating artery aneurysm surgery. This vessel usually arises from the A2 segment of the ACA, immediately after the Anterior Communicating Artery. References: Loukas M, Louis RG Jr, Childs RS. Anatomical examination of the recurrent artery of Heubner. Clin Anat. 2006 Jan; 19(1):25-31; Falougy et al., Biomed Res Int 2013 PMID 23936853
29
The recurrent artery of Heubner most frequently arises from the Answers: A. A1 segment of the anterior cerebral artery B. Ophthalmic segment of the internal cerebral artery C. Communicating segment of the anterior cerebral artery D. Communicating segment of the internal cerebral artery E. A2 segment of the anterior cerebral artery
A2 segment of the anterior cerebral artery ## Footnote Discussion: The recurrent artery of Heubner most commonly arises from the post-communicating segment of the anterior cerebral artery (A2) in 47.81% of cases, from the communicating segment in 43.4% of cases, and from the pre-communicating segment in only 3.55% of cases. References: Falougy et al., Biomed Res Int 2013 PMID 23936853. Loukas M, Louis RG, Jr., Childs RS. Anatomical examination of the recurrent artery of Heubner. Clin Anat. Jan 2006;19(1):25-31
30
From which of the following arteries does the tentorial artery originate? Answers: A. Meningohypophyseal trunk B. Petrous internal carotid C. Maxillary artery D. Middle meningeal artery E. Middle cerebral artery
Meningohypophyseal trunk ## Footnote Discussion: The tentorial artery arises from the meningohypophyseal trunk and supplies the mass of the tentorium and its petrous attachment. The meningohypophyseal trunk arises from the posterior genu of the cavernous internal carotid. References: Yasargil. Microneurosurgery. Vol 1; Osborn AG. Diagnostic Neuroradiology: A text/Atlas. 1st ed. Philadelphia: Mosby-Elsevier; 1994
31
The artery that appears to be the primary contributor to the tumor “blush” illustrated on the lateral carotid angiogram shown also typically supplies blood to which of the following brain structures? Answers: A. cerebellar vermis B. internal capsule C. medial lemniscus D. midbrain E. lateral lemniscus
internal capsule ## Footnote Discussion: The anterior choroidal artery is depicted this lateral carotid angiogram. It supplies the posterior limb of the internal capsule, the optic tract/radiations and globus pallidus internus. Damage to this artery results in contralateral hemiplegia, contralateral hemianesthesia and homonimous hemianopsia. References: Radiographia.org. Accessed August 20, 2016.Krayenbu¨hl H, Yas¸argil MG, eds. Cerebral angiography. 2nd ed. Philadelphia, PA: Lippincott, Williams & Wilkins, 1968; Hupperts, R. M. M., et al. "Infarcts in the anterior choroidal artery territory Anatomical distribution, clinical syndromes, presumed pathogenesis and early outcome." Brain 117.4 (1994): 825-834
32
A 56-year-old man undergoes surgical clipping of a left posterior communicating artery aneurysm. The patient awakens with a contralateral hemiparesis and contralateral visual field loss. Compromise of which of the following vessels is implicated in this patient’s new deficit? Answers: A. Artery of Huebner B. posterior communicating artery C. Lenticulostriate artery D. Anterior choroidal artery E. Tentorial artery
Anterior choroidal artery ## Footnote Discussion: The Anterior Choroidal Artery supplies the posterior limb of the internal capsule, the optic tract/radiations and globus pallidus internus. Damage to this artery results in contralateral hemiplegia, contralateral hemianesthesia and homonimous hemianopsia. References: Afifi A, Bergman R, eds. Functional Anatomy: Text and Atlas. 2nd ed. New York, NY: McGraw-Hill Education; 2005:362; Hupperts, R. M. M., et al. "Infarcts in the anterior choroidal artery territory Anatomical distribution, clinical syndromes, presumed pathogenesis and early outcome." Brain 117.4 (1994): 825-834
33
Which of the following structures is supplied predominantly by extradural branches of the internal carotid artery? Answers: A. Maxillary branch (CN V2) B. Tentorium cerebelli C. Hypothalamus D. Abducens nerve E. Oculomotor nerve
Tentorium cerebelli ## Footnote Discussion: The tentorium cerebelli is supplied by the Artery of Bernasconi and Cassinari, the Italian Artery. Tentorial meningioms and arterovenous malformations are typically supplied by this branch. References: Harris FS, Rhoton AL Jr. Microsurgical anatomy of the cavernous sinus: A microsurgical study. J Neurosurg. 1976;45:169-80; Osborn AG. Diagnostic Neuroradiology: A text/Atlas. 1st ed. Philadelphia: Mosby-Elsevier 1994
34
The persistent trigeminal artery connects the basilar artery with which of the following arteries? Answers: A. Petrous Internal Carotid Artery B. Posterior communicating artery C. Cavernous Internal Carotid Artery D. Cervical Internal Carotid Artery E. External Carotid Artery
Cavernous Internal Carotid Artery ## Footnote Discussion: The persistent trigeminal artery arises from the cavernous segment of the internal carotid artery and courses to the basilar artery. It is present in 0.1 to 0.5 percent of individuals and is the most common persistent fetal intracranial artery in adults. References: Carpenter MB, ed. Core text of neuroanatomy. 4th ed. Baltimore, MD: Lippincott, Williams & Wilkins, 1991; Osborn AG. Diagnostic Neuroradiology: A text/Atlas. 1st ed. Philadelphia: MosbyElsevier 1994
35
The anterior choroidal artery most likely supplies which of the following structures? Answers: A. Gyrus rectus B. Lateral geniculate nucleus C. Habenula D. Anterior insula E. Anterior limb of the internal capsule
Lateral geniculate nucleus ## Footnote Discussion: The anterior choroidal artery most often arises from the internal carotid artery distal to the posterior communicating artery. It supplies the posterior limb of the internal capsule, the optic tract, the lateral geniculate nucleus, the medial temporal lobe, and the globus pallidus pars interna (GPi). Ischemia of its territory classically results in contralateral hemisensory loss, hemiparesis, and hemianopia. References: Ois et al., J Neurological Sciences 2009 PMID 19324377; Hupperts, R. M. M., et al. "Infarcts in the anterior choroidal artery territory Anatomical distribution, clinical syndromes, presumed pathogenesis and early outcome." Brain 117.4 (1994): 825-834
36
The arrow on the sagittal MR image shown is pointing to which of the following structures? Answers: A. Vein of galen B. Internal cerebral vein C. Vein of Trolard D. Inferior sagittal sinus E. Thalamostriate vein
Internal cerebral vein ## Footnote Discussion: The arrow points to the internal cerebral vein. The internal cerebral veins are paired and run in the roof of the third ventricle between the two leaves of the velum interpositum. The basal veins of rosenthal join with the internal cerebral veins to create the Vein of Galen. The thalamostriate vein commences in the groove between the corpus striatum and thalamus and unites with a number of veins behind the crus of the fornix with the choroid vein to form the internal cerebral vein. The vein of Trolard is formed by a number of parietal veins to drain into the superior sagittal sinus. References: Osborn AG, ed. Diagnostic Cerebral Angiography. 2nd ed. Philadelphia, PA: Lippincott, Williams & Wilkins, 1999; Krayenbühl H, Yaşargil MG, Huber P et-al. Cerebral angiography. Thieme Publishing Group. (1982) ISBN:3136125029
37
A patient undergoes a right pterional craniotomy for clipping of an aneurysm. A postoperative CT scan of the head is shown. This finding is most likely related to injury of which the following structures? Answers: A. Anterior cerebral artery B. Posterior communicating artery C. Anterior choroidal artery D. Recurrent artery of Heubner E. Superior Hypophyseal Artery
Recurrent artery of Heubner ## Footnote Discussion: CT scan demonstrates infarction of the caudate head, anterior portion of the lentiform nucleus, and anterior limb of the internal capsule. These structures are supplied by the recurrent artery of Heubner which can be injured during anterior communicating artery aneurysm surgery. This vessel usually arises from the A2 segment of the ACA, immediately after the Anterior Communicating Artery. References: Calis M, Oz Z, Isikay I, et al. An extremely rare complication following frontoorbital advancement: infarction of the recurrent artery of Heubner. Childs Nerv Syst. 2017 Jan;33(1):197-199. Loukas M, Louis RG, Jr., Childs RS. Anatomical examination of the recurrent artery of Heubner. Clin Anat. Jan 2006;19(1):25-31
38
A 59-year-old man is evaluated for right-sided hemisensory loss. Physical examination shows no additional abnormalities. Which of the following vessels is most likely occluded? Answers: A. Left middle cerebral artery B. Right posterior cerebral artery C. Left lateral posterior choroidal artery D. Right superior cerebellar artery E. Left anterior cerebral artery
Left lateral posterior choroidal artery ## Footnote Discussion: Sensory loss is most likely due to an infarct of the ventral posterior nucleus of the thalamus which is supplied by the lateral posterior choroidal artery. References: Galloway JR, Greitz T. The medial and lateral choroid arteries. An anatomic and roentgenographic study. Acta radiol. 1960 May;53:353-366; Neau, J.-P., & Bogousslavsky, J. (1996). The syndrome of posterior choroidal artery territory infarction. Annals of Neurology, 39(6), 779–788. doi:10.1002/ana.410390614
39
Which of the following arteries supplies the descending spinal nucleus of the trigeminal nerve? Answers: A. Posterior inferior cerebellar artery B. Posterior communicating artery C. Superior cerebellar artery D. Vertebral artery E. Anterior inferior cerebellar artery
Posterior inferior cerebellar artery ## Footnote Discussion: Posterior inferior cerebellar artery (PICA) occlusion causes lateral medullary syndrome, also known as Wallenberg syndrome. One aspect of this syndrome is ipsilateral loss of facial sensation secondary to involvement of the spinal nucleus of the trigeminal nerve. Other findings in this syndrome include contralateral loss of pain and temperature sense in the body, decreased gag and taste, dysphagia and hoarseness, Horner’s syndrome, and cerebellar ataxia from involvement of cranial nerves IX and X, the lateral spinothalamic tract, the descending sympathetic fibers, and the inferior cerebellar peduncle. References: Citow J, Macdonald R, Puffer R et al., Comprehensive Neurosurgery Board Review, ed. 3rd Edition. Thieme; 2019. doi:10.1055/b-005-148973; Carpenter MB. Core Text of Neuroanatomy. 4th ed. Baltimore: Williams and Wilkins; 1991: 453
40
Which of the following arteries arises from the cavernous segment of the internal carotid artery? Answers: A. Anterior choroidal artery B. Otic artery C. Caroticotympanic artery D. Superior hypophyseal artery E. Tentorial artery
Tentorial artery ## Footnote Discussion: The cavernous ICA normally emits 3 major arteries. The meningohypophyseal trunk, which arises from the posterior genu of the cavernous ICA, subsequently divides into the tentorial artery, the inferior hypophyseal artery, and the dorsal meningeal artery. The second artery consists of the inferolateral trunk, which supplies the cranial nerves that traverse the cavernous sinus. Finally, the cavernous ICA frequently emits capsular arteries of McConnell, which contribute to the vascular supply of the pituitary gland. Importantly, the cavernous ICA can also emit the anomalous trigeminal artery, which represents a persistent carotico-basilar anastomosis. References: Tran-Dinh, Neurosurgery 1987 PMID 3561725; Osborn AG. Diagnostic Neuroradiology: A text/Atlas. 1st ed. Philadelphia: Mosby-Elsevier; 1994.
41
Which of the following veins is formed by fusion of the septal and thalamostriate veins? Answers: A. Vein of Galen B. Basal vein of Rosenthal C. Anterior caudate vein D. Superior choroidal vein E. Internal cerebral vein
Internal cerebral vein ## Footnote Discussion: The fusion of the septal and thalamostriate veins gives rise to the internal cerebral vein. The bilateral internal cerebral veins and bilateral basal veins of Rosenthal join to form the vein of Galen. The vein of Galen drains to the straight sinus at its junction with the inferior sagittal sinus. The anterior caudate vein drains into the thalamostriate vein. The septal vein can be sacrificed at surgery if needed, the others cannot. References: Kendall & Schwartz. Principles of Neural Science. 4th ed; Ono et al., Neurosurgery 1984 PMID 6504279; Osborn AG. Diagnostic Neuroradiology: A text/Atlas. 1st ed. Philadelphia: MosbyElsevier; 1994.
42
During a retrosigmoid approach for a small vestibular schwannoma resection, which of the following arteries is at greatest risk for injury? Answers: A. Posterior Inferior Cerebellar Artery B. Anterior Inferior Cerebellar Artery C. P1 segment of the Posterior Cerebral Artery D. Superior Cerebellar Artery E. posterior communicating artery
Anterior Inferior Cerebellar Artery ## Footnote Discussion: The AICA, as well as the Labyrinthine artery (most commonly a branch of the AICA) are at risk during a retrosigmoid approach for schwannoma resection. The labyrinthine artery courses into the internal auditory meatus and damage to this artery can lead to hearing loss. References: Rhoton A. Cranial Anatomy and Surgical Approaches. Neurosurgery. 2003; 53(3): 545; Koos WT, Spetzler RF, Lang J. Color Atlas of Micro-neurosurgery: Microanatomy Approaches and Techniques. 2nd ed. Vol 1. Thieme; 1993: 514
43
A 25-year-old man is evaluated because of a one-month history of headache and unsteady gait. An MR image and a CT scan are shown. This patient’s abnormality is most likely in which of the following vascular distributions? Answers: A. internal carotid artery B. posterior communicating artery C. posterior cerebral artery D. anterior cerebral artery E. posterior inferior cerebellar artery
posterior cerebral artery ## Footnote Discussion: A choroid plexus papilloma is seen in the left lateral ventricle. The choroid of the lateral ventricles is supplied by the posterior choroidal artery arising from the posterior cerebral artery. The medial posterior choroidal artery supplies the choroid plexus of the third ventricle. The lateral posterior choroidal artery supplies the choroid plexus of the lateral ventricles. References: Osborn AG. Diagnostic Neuroradiology: A text/Atlas. 1st ed. Philadelphia: Mosby-Elsevier 1994; James R. Galloway & Torgny Greitz (1960) The medial and lateral choroid arteries, Acta Radiologica, 53:5, 353-366, DOI: 10.3109/00016926009171684
44
Which of the following structures is indicated by the arrows in the dissection shown? Answers: A. Transverse sinus B. Superior petrosal sinus C. Straight sinus D. Inferior petrosal sinus E. Cavernous sinus
Inferior petrosal sinus ## Footnote Discussion: The inferior petrosal sinus can be seen in its course from the cavernous sinus to drain into the internal jugular vein via the jugular foramen. References: Rhoton AL, Apuzzo MLJ, eds. Cranial Anatomy and Surgical Approaches. 1st ed. Lippincott Williams & Wilkins, 2003; Osborn AG, Slazman KL, Jhaveri MD, et al, eds. Diagnostic Imaging: Brain 3e. 3rd ed. Elsevier, 2015
45
Which of the following structures is indicated by the arrow in the photograph shown? Answers: A. ophthalmic artery B. anterior choroidal artery C. Anterior inferior cerebral artery D. Posterior Communicating Artery E. superior cerebellar artery
anterior choroidal artery ## Footnote Discussion: The anterior choroidal artery exits the ICA just distal to the origin of the posterior communicating artery. References: Cooper IS. Ligation of the anterior choroidal artery for involuntary movements of parkinsonism. Psychiatr Q. 1953 Apr;27(2):317-9; Hupperts, R. M. M., et al. "Infarcts in the anterior choroidal artery territory Anatomical distribution, clinical syndromes, presumed pathogenesis and early outcome." Brain 117.4 (1994): 825-834
46
Which of the following structures of the brain is supplied by the recurrent Heubner's artery? Answers: A. Internal capsule B. Uncus C. External capsule D. Thalamus E. Hypothalamus
Internal capsule ## Footnote Discussion: The internal capsule, parts of the putamen and caudate and the septal nuclei are supplied by the recurrent artery of Heubner. This vessel usually arises from the A2 segment of the ACA, immediately after the Anterior Communicating Artery. References: Falougy et al., Biomed Res Int 2013 PMID 23936853. Loukas M, Louis RG, Jr., Childs RS. Anatomical examination of the recurrent artery of Heubner. Clin Anat. Jan 2006;19(1):25-31
47
In the angiogram shown, which of the following vascular structures appears to be abnormally dilated? Answers: A. Middle meningeal artery B. Ascending pharyngeal artery C. Superior hypophyseal artery D. Caroticotypmpanic branch E. Meningohypophyseal trunk
Meningohypophyseal trunk ## Footnote Discussion: The meningohypophyseal trunk arises from the distal genu of the cavernous carotid artery and supplies portions of the pituitary gland, tentorium, and clivus. It has important ICA–ECA anastomoses through the ascending pharyngeal and middle meningeal arteries. The three major branches of the MHT are the artery of Bernasconi and Cassinari, inferior hypophseal artery and clival dural branches. The MHT may be enlarged in pathological conditions, for example, tentorial meningiomas and dural arteriovenous fistulas. References: Lasjaunias P, Berenstein A, Ter Brugge KG, eds. Surgical Neuro-Angiography, Volume 1. 2nd ed. Springer, 2001; Barrow, D. L., Spector, R. H., Braun, I. F., Landman, J. A., Tindall, S. C., & Tindall, G. T. (1985). Classification and treatment of spontaneous carotid-cavernous sinus fistulas. Journal of Neurosurgery, 62(2), 248–256. doi:10.3171/jns.1985.62.2.0248
48
Which of the following is the most common primitive/persistent fetal intracranial artery in adults? Answers: A. Persistent hypoglossal artery B. Persistent otic artery C. Persistent trigeminal artery D. Primitive prepontine artery E. Proatlantal intersegmental artery
Persistent trigeminal artery ## Footnote Discussion: The persistent fetal cranial arteries that may be seen in the adult include - in order from most to least common - the persistent trigeminal artery, the persistent hypoglossal artery, the proatlantal intersegmental artery, and the persistent otic artery. The persistent trigeminal artery is seen in between 0.1% and 0.5% of individuals. References: Tubbs RS, Verma K, Riech S, et al. Persistent fetal intracranial arteries: a comprehensive review of anatomical and clinical significance. J Neurosurg. 2011 Apr;114(4):1127-34; Osborn AG, Slazman KL, Jhaveri MD, et al, eds. Diagnostic Imaging: Brain 3e. 3rd ed. Elsevier, 2015
49
Which of the following arterial branches usually supply low-flow carotid cavernous fistulas? Answers: A. Posterior communicating artery B. Recurrent artery of Heubner C. Ophthalmic artery D. Internal maxillary artery E. Anterior choroidal artery
Internal maxillary artery ## Footnote Discussion: The internal maxillary artery usually supplies low-flow carotid cavernous fistulas. Indirect, low-flow carotid cavernous fistulas (CCFs) are categorized into type B, C, and D. Type B CCFs arise from meningeal branches of the internal carotid artery (ICA), Type C CCFs arise from meningeal branches of the external carotid artery (ECA), and Type D CCFs arise from meningeal branches of the ICA and ECA. Of the choices listed, the internal maxillary artery is the only meningeal branch of the ICA or ECA. References: Ellis & Goldstein. “Carotid-cavernous fistulas.” Neurosurg Focus, 2012. PMID: 22537135; Barrow, D. L., Spector, R. H., Braun, I. F., Landman, J. A., Tindall, S. C., & Tindall, G. T. (1985). Classification and treatment of spontaneous carotid-cavernous sinus fistulas. Journal of Neurosurgery, 62(2), 248–256. doi:10.3171/jns.1985.62.2.0248
50
A 65-year-old woman is evaluated after undergoing intracranial anterior cerebral artery bypass and trapping of a fusiform anterior communicating artery aneurysm. Postoperatively, MR imaging of the brain reveals diffusion restriction in bilateral anterior internal capsules. This patient is most likely to have which of the following neurological deficits? Answers: A. Hemihypestensia B. Hemianopsia C. Hemineglect D. Paraplegia E. Akinetic mutism
Akinetic mutism ## Footnote Discussion: Akinetic mutism is a disorder of consciousness characterized by unresponsiveness but with the superficial appearance of alertness. The patient's eyes are open and he may seem to look at the examiner but he neither speaks nor moves, nor is the examiner able to communicate with the patient. Bilateral lesions of the head of the caudate nucleus associated with destruction of medial putamen, septum, medial frontal cortex, and cingulate cortex can result in akinetic mutism. This could arise from compromise of the bilateral recurrent arteries of Huebner. References: Uzun I, Gurdal E, Cakmak YO, Ozdogmus O, Cavdar S. A reminder of the anatomy of the recurrent artery of heubner. Cent Eur Neurosurg. 2009 Feb;70(1):36-8. Toyoda K. Anterior cerebral artery and Heubner's artery territory infarction. Front Neurol Neurosci. 2012;30:120-2. Freemon 1971. http://dx.doi.org/10.1136/jnnp.34.6.693
51
A 34-year-old female presents with spontaneous proptosis and chemosis of the left eye. Cerebral angiography (left common carotid injection) is shown . What is the best management option? A. Optic nerve sheath fenestration B. Ventriculoperitoneal shunt C. Transarterial carotid sacrifice D. Transvenous embolization E. Craniotomy
D. Transvenous embolization Discussion: The patient has an indirect carotid-cavernous fistula (CCF). The first line treatment is transvenous embolization of the cavernous sinus with a high cure rate. Transarterial carotid sacrifice is occasionally necessary for direct CCF that cannot be cured with transvenous embolization. Craniotomy, optic nerve sheath fenestration, and ventriculoperitoneal shunt are not appropriate treatment options.
52
According to AHA guidelines for early ischemic stroke management, IV tPA should be administered within what time frame after the onset of symptoms? A. 2.5 hours B. 5.5 hours C. 4.5 hours D. 3.5 hours E. 1.5 hours
C. 4.5 hours Discussion: According to the AHA guidelines tPA should be given within 4.5 hours after the onset of symptoms of an ischemic stroke if no contraindication for its administration exist.
53
What is the most likely mechanism underlying an intracranial hemorrhage associated with an intracranial dural fistula? A. Arterial aneurysm rupture B. Increased intracranial pressure C. Hyperemia D. Venous hypertension E. Brain edema
D. Venous hypertension Discussion: Patients with DAVFs may be completely asymptomatic. Symptoms, when present, may range from mild symptoms to fatal hemorrhage. Symptoms of DAVFs may be characterized further as either nonaggressive (e.g., tinnitus) or aggressive (e.g., intracerebral, subarachnoid, or subdural hemorrhage and neurologic deficits). These aggressive features are usually due to venous hypertension.
54
In patients who receive medical therapy alone for asymptomatic carotid stenosis of 60-99%, what is the 5-year risk of stroke or death? A. 17% B. 23% C. 29% D. 5% E. 11%
E. 11% Discussion: Explanation: The endarterectomy for asymptomatic carotid artery stenosis (ACAS) study showed that asymptomatic patients who have a degree of carotid stenosis between 60% to 99% and who are on medical therapy alone have a 5 year risk of stroke or death of 11% as compared to similar patients who receive both medical therapy and carotid endarterectomy (5.1%).
55
A 54 year old male presented with a ruptured anterior communicating artery aneurysm. The patient underwent surgical repair. On postoperative day 5, the patient complained of mild weakness in his left leg. Transcranial Doppler showed moderate vasospasm of the right anterior cerebral artery. What is the most appropriate initial management? A. Statins B. Angioplasty C. Hyperdynamic therapy D. nimodipine E. Intraarterial verapamil injection
C. Hyperdynamic therapy Discussion: The combination of induced hyperdynamic therapy (hypertension and hypervolemia --triple-H therapy/ hemodilution is no longer actively pursued) is often utilized as a first step to treat cerebral vasospasm after aneurysmal subarachnoid hemorrhage. Angioplasty is reserved for patients who remain symptomatic despite triple H therapy. While statins may improve overall outcome after SAH, they do not have a demonstrated role in addressing acute ischemia from vasospasm. Nimodipine has been shown to be beneficial as a prophylactic agent for vasospasm, but not a treatment for acute vasospasm.
56
During intraoperative aneurysm rupture, which medication can help achieve temporary flow arrest? A. Nimodipine B. Vitamin K C. Labetalol D. Adenosine E. Protamine
D. Adenosine Discussion: During intraoperative rupture of aneurysms, adenosine can be safely used to induce a short reversible hypotension and temporary asystole to decrease the bleeding from the ruptured aneurysm. This allows better visualization and control and limited time (30-45s) to place temporary clips . Protamine is used to antagonize heparin. Vitamin K is used to reverse anticoagulation from warfarin. Labetalol and nimodipine have no effect on bleeding.
57
A 28-year-old male presents with acute onset facial asymmetry, diplopia, and hemifacial numbness. The patient’s MRI is shown. What is the molecular mechanism for disease pathogenesis? A. Activation of MEKK3 signaling B. Activation of ERK/MAPK signaling C. Mutation of BRAF V600 E D. Activation of AKT1 signaling E. Activation of SMO signaling
A. Activation of MEKK3 signaling Discussion: The depicted lesion is a cavernous malformation. MEKK3 signaling targets genes Klf2 and Klf4 and is increased in the endothelial cells of cavernous malformations. It is believed to be the causal mechanism for disease pathogenesis. In an age of precision medicine, the genetic and molecular basis of tumors and vascular lesions are poised to change the classification systems used to describe many central nervous system pathologies. Activating mutations in SMO and AKT1 have been recently implicated in meningioma pathogenesis. BRAF V600E, a key activator of ERK/MAPK signaling, is the principle oncogenic driver in papillary craniopharyngioma.
58
What is the most common presentation of an adult patient with moyamoya disease? A. Intracranial hemorrhage B. Ischemic infarcts/transient ischemic attacks C. Gait disturbances D. Dystonia E. Seizure
A. Intracranial hemorrhage Discussion: Adults with moyamoya disease commonly present with evidence of intracranial hemorrhage (intracerebral or interaventricular). Children, on the other hand, present typically with recurrent infarcts or transient ischemic attacks. Fine involuntary movements of the extremities and slowly progressive mental impairment have also been commonly observed in children. Dystonia can be a presenting symptom more common in the pediatric moyamoya patient population.
59
A 61-year-old man presents with confusion. Susceptibility-weighted MR imaging demonstrates multiple abnormalities (figure). What is the most likely diagnosis? A. CADASIL (cerebral autosomal dominant arteriopathy with subcortical infarcts and leukoencephalopathy) B. Leptomeningeal carcinomatosis C. Hypertensive hemorrhages D. Cerebral amyloid angiopathy E. Creutzfeldt-Jakob disease
D. Cerebral amyloid angiopathy Discussion: The correct answer is amyloid angiopathy. The gradient echo images are sensitive to blood products, which will appear hypointense. On the left image, left frontal and left parietal superficial siderosis is seen. The right image shows multiple cortical and subcortical microhemorrhages. Superficial siderosis results from degraded blood products within the cortical gyral sub-pial space. Blood in this distribution can result from amyloid-related microhemorrhages. Lobar microhemorrhages with superficial siderosis are not characteristic of CADASIL, carcinomatosis, or Creutzfeldt-Jakob disease. Hypertensive hemorrhages tend to occur in deeper brain regions such as the basal ganglia and brain stem.
60
A 55-year-old man presents to the emergency department with an acute onset of right hemiplegia that occurred 30 minutes prior. CT scan is negative for hemorrhage. What factor would contraindicate the administration of IV tPA? A. INR of 1.5 B. Major surgery 12 months ago C. Past medical history of bleeding stomach ulcer 2 years ago D. Refractory hypertension (SBP>180mmHg) E. Severe Symptoms (NIHSS>8)
D. Refractory hypertension (SBP>180mmHg) Discussion: Absolute contraindications to intravenous tPA for acute cerebral infarction are based on safety guidelines as prescribed by the FDA for alteplase. Among these are recent surgery/trauma (less than 15 days), recent intracranial or spinal surgery, head trauma, or stroke (less than 3 months), seizure at onset of stroke symptoms, oral anticoagulant use with INR > 1.7, and a platelet count < 100,000 (thrombocytopenia). Relative contraindications to tPA include minor or rapidly improving symptoms, major surgery or serious non-head trauma in the previous 14 days, history of GI or urinary tract hemorrhage within 21 days, recent arterial puncture at noncompressible site, recent lumbar puncture, post myocardial infarction pericarditis, and pregnancy.
61
What factor is most predictive of postoperative intracerebral hemorhage after carotid endarterectomy? A. Young age B. Calcified plaque C. Female Sex D. Cerebral hypoperfusion E. Length of stenosis
D. Cerebral hypoperfusion Discussion: Intracranial hemorrhage (ICH) following carotid endarterectomy (CEA) typically occurs within the first week, in the identifiable settings of hyperperfusion, a perioperative cerebral ischemic event, and anticoagulation therapy. Other risk factors for developing ICH after carotid endarterectomy include advanced age, associated hypertension, presence of high grade stenosis, poor collateral flow and slow flow in the MCA territory in angiography. The latter angiographic evidence is the most important predictive factor. Perioperative cerebral ischemia can result in intracerebral hemorrhage after CEA even in the absence of cerebral hyperperfusion and/or anticoagulation therapy. Further, cerebral perfusion imaging performed immediately after CEA is a useful modality for the identification of occult cerebral ischemia or hyperperfusion that may lead to intracerebral hemorrhage.
62
During a pterional exposure of the sylvian fissure, you encounter significant brain swelling. In order to achieve rapid brain relaxation, identify the most appropriate point on the associated figure through which to place a ventriculosotomy and access the frontal horn of the lateral ventricle. A. 3 B. 5 C. 4 D. 1 E. 2
E. 2 Discussion: The neurosurgeon should have a see-through type awareness of the position of the deep structures in relation to the cortical surface. The frontal horn is located deep to the inferior frontal gyrus (2), the atrium is located deep to the supramarginal gyrus (3), and the temporal horn is located deep to the middle temporal gyrus (6). The forntal horn could also be cannulated through the middle frontal gyrus (1) with a catheter oriented perpendicular to the cortical surface. This area however, would not be typically exposed in a pterional approach.
63
What is the mortality rate of a MCA infarction with malignant cerebral edema managed with intensive non-operative care? A. 80% B. 40% C. 20% D. 0% E. 60%
A. 80% Discussion: The mortality rate of malignant MCA infarction with space occupying lesion under conservative management is estimated to be 80%. Mortality has been shown to decrease with hemicraniectomy for malignant MCA infarction. Metaanalysis from 3 randomized control trials (DECIMAL, DESTINY, HAMLET) found that hemicraniectomy performed within 48 hours decreases mortality and increases the number of patients with favorable functional outcomes.
64
A 60 year old female presents with sudden onset of vertigo, headache, vomiting and imbalance. Her imaging (after 2 days) is shown. What is the cause of the patient’s symptoms? A. Cerebellar astrocytoma B. Cerebellar infarction C. Cerebellar abscess E. Lhermitte-Duclos disease E. Multiple sclerosis
B. Cerebellar infarction Discussion: The sudden onset of symptoms and MRI findings are consistent with a vascular pathology. The MRI shows a left cerebellar stroke in the PICA territory with compression of the fourth ventricle without evidence of hydrocephalus. The other listed options do not have both the same imaging appearance and symptoms. Lhermitte-Duclos disease (LDD) is also known as dysplastic cerebellar gangliocytoma which is a rare tumor of the cerebellum.
65
Which vein is indicated by the arrow in the figure? A. Vein of Trolard B. Vein of Galen C. Basal vein of Rosenthal C. Internal Cerebral vein D. Thalmostriate vein
C. Basal vein of Rosenthal Discussion: The arrow points to the Basal vein of Rosenthal, which joins with the internal cerebral vein to create the Vein of Galen. The thalamostriate vein commences in the groove between the corpus striatum and thalamus and unites with a number of veins behind the crus of the fornix with the choroid vein to form the internal cerebral vein. The vein of Trolard is formed by a number of parietal veins to drain into the superior sagittal sinus. The internal cerebral vein runs in the roof of the third ventricle between the two leaves of the velum interpositum.
66
Based on the ISUIA study, what is the 5-year cumulative rupture risk of an unruptured 5mm middle cerebral artery aneurysm? A. 0% B. 10% C. 7.5% D. 5% E. 2.5%
A. 0% Discussion: The international study of unruptured intracranial aneurysms (ISUIA) is the largest prospective study to provide data on cumulative rupture risk. In this cohort, isolated unruptured aneurysms less than 7mm in the anterior circulation have essentially a 0% 5-year cumulative risk of rupture.
67
A 43-year old patient experienced a visual field defect and hemisensory loss after clipping of a PCA aneurysm. What is the most likely site of injury? A. Superior cerebellar artery B. Posterior communicating artery C. Superior hypophyseal artery D. Lateral posterior choroidal artery E. Anterior choroidal artery
D. Lateral posterior choroidal artery Discussion: Posterior choroidal artery territory infarcts may lead to damage of the lateral geniculate body, pulvinar, posterior thalamus, hippocampus and parahippocampal gyrus. Clinical symptoms would include visual field defects, hemisensory disturbances, neuropsychological dysfunctions and abnormal movements, and rarely hemiparesis and eye movement disorders. Infarctions at the level of the posterior communicating artery generally causes visual and sensory symptoms. Occlusion of the anterior choroidal artery causes a triad of hemiparesis, homonymous hemianopsia and hemihypesthesia. The superior cerebellar artery supplies the cerebellar vermis and the superior cerebellum and thus will not lead to the above symptoms. The superior hypophyseal artery originates from the ICA and supplies the optic apparatus and the pituitary infundibulum.
68
A 44 year old man with a history of blunt head trauma 4 days earlier presents with a progressively pulsatile exophthalmus, headache, and double vision. A CT head performed after his initial trauma was normal. What is the most appropriate treatment for this condition? A. Delayed endovascular repair B. Radiation C. Cranitomy for open repair D. Urgent endovascular repair E. A trial of manual direct compressoin of cervical carotid 3-4x daily
D. Urgent endovascular repair Discussion: This patient most likely has a direct, high flow carotid-cavernous fistula (CCF). After confirming the diagnosis with a cerebral angiogram, endovascular repair via transarterial or transvenous routes is the first line treatment with ~90% cure rate. Improvement in symptoms depends on their severity and duration at the time of repair. For direct, high flow lesions, urgent repair is indicated while for indirect low flow lesions repair may be undertaken more conservatively. Historically, patients with indirect CCFs were managed with manual direct compression that had a low cure rate. Craniotomy is indicated if endovascular interventions have been unsuccessful. Some success has been reported with radiotherapy for indirect fistulas; however, endovascular interventions remain the first line approach.
69
A 60 year old patient presents to the emergency department with a history suggestive of acute ischemic stroke. What is the best brain imaging modality to confirm the diagnosis? A. Digital Subtraction angiography B. Noncontrast CT scan C. CT angiography D. PET scan E. Magnetic resonance imaging
E. Magnetic resonance imaging Discussion: According to the latest stroke guidelines, the most reliable diagnostic modality that shows the ischemic stroke is MRI including diffusion weighted imaging (DWI). CT scan should be done if MRI is not available; however MRI is superior (Class I level of evidence B). Furthermore, it is not recommended to perform invasive imaging studies on patients with TIAs as the first step of management (Class I , level of evidence A). Thus the patient does not need to have a DSA. Non-invasive imaging like CTA or MRA is recommended to exclude the presence of intracranial stenosis (Class I, level of evidence A). Recent studies focusing on penumbral biomarkers promise to individualize treatment windows in acute ischemic stroke. A novel magnetic resonance imaging approach that measures oxygen metabolic index (OMI), a parameter closely related to positron emission tomography (PET)-derived cerebral metabolic rate of oxygen utilization (CMRO2) derives a pair of ischemic thresholds: (1) an irreversible-injury threshold that differentiates ischemic core from penumbra and (2) a reversible-injury threshold that differentiates penumbra from tissue not-at-risk for infarction.
70
30 year old female, 2 weeks post-partum, presents to the ED after 4 hours with acute onset right hemiplegia, lethargy and left gaze deviation. Her NIHSS score is 24. Her angiogram is shown. What is the best management option? A. IV tPA B. EC/IC bypass C. Aspirin D. IV abciximab E. Thrombectomy with stent-retrieval
E. Thrombectomy with stent-retrieval Discussion: The MRCLEAN trial showed that in patients with acute ischemic stroke caused by a proximal intracranial occlusion of the anterior circulation, intraarterial treatment performed within 6 hours after stroke onset was effective and safe. The trial enrolled 500 patients (233 assigned to intraarterial treatment and 267 to usual care alone who were treated with intravenous alteplase before randomization (to receive intraarterial treatment or not). Retrievable stents were used in 190 of the 233 patients (81.5%) assigned to intraarterial treatment. There was an absolute difference of 13.5 percentage points (95% CI, 5.9 to 21.2) in the rate of functional independence (modified Rankin score, 0 to 2) in favor of the intervention (32.6% vs. 19.1%). There were no significant differences in mortality or the occurrence of symptomatic intracerebral hemorrhage.
71
A 65 year old woman on warfarin for her chronic atrial fibrillation, has been diagnosed with an intracerebral hemorrhage after presenting to the emergency room. Her INR was found to be 6. What is the best next step in the management of the patient? A. Administer cryoprecipitate B. Observe the patient closely C. Administer Factor VIII D. Ultrasound of the liver and liver function tests E. Administer Fresh Frozen Plasma
E. Administer Fresh Frozen Plasma Discussion: Fresh frozen plasma (FFP), along with vitamin K, has been the mainstay of treatment for the rapid correction of the international normalized ration (INR) for oral anticoagulants related intracranial hemorrhage. According to the latest ICH guidelines, patients who present with ICH and whose INR is elevated due to oral anticoagulation should have their warfarin withheld, receive therapy to replace the vitamin-K dependent coagulation factors blood coagulation factors, and receive Vitamin K intravenously (Class I; Level of Evidence C). Checking the function of the liver is not necessary in this case because the cause of bleeding is most likely due to the medication and not to liver dysfunction. Platelet deficiency or platelet dysfunction would not affect the INR. Cryoprecipitate is no longer used as the current standard for factor VIII deficiency in the U.S., however factor VIII is not vitamin K dependent.
72
A 44 year old man presents with seizure, headache, and left visual field cut. MRI is shown. What is the most likely benefit of embolization for this lesion? A. Complete obliteration of the AVM B. Decreased risk of seizures C. Improvement of vision D. Decreased risk of rupture before further therapy E. Reducing AVM volume for subsequent treatment
E. Reducing AVM volume for subsequent treatment Discussion: The benefit of embolization of large AVMs is to decrease the lesion size in conjunction with surgical or radiosurgical therapy. Incomplete embolization will not lower the risk of rupture and may even increase the rates of rupture in the interim. The likelihood of complete obliteration of large AVMs is low, except in small lesions with few vascular pedicles. There is no significant difference in the incidence of seizures. Visual outcomes are unlikely to be affected by embolization alone.
73
On this lateral projection angiogram, the indicated artery supplies the: A. Dura B. Scalp C. Superior parietal lobule D. Cingulate gyrus E. Pre-central gyrus
A. Dura Discussion: The correct answer is the dura. The indicated artery is part of the middle meningeal arterial system. These arteries can often be differentiated from scalp or cerebral arteries because their trajectories are typically more linear. Tracing the indicated artery back to its origin shows that this meningeal artery originates from the ophthalmic artery, a known anatomic variant, rather than its typical origin off of the internal maxillary artery as one of the terminal branches of the external carotid arterial tree. Because it is a meningeal artery, it typically does not provide blood supply to the scalp or cortex, except in some pathologic situations, such as dural arteriovenous fistulae, AVMs, and moyamoya disease.
74
What is the main arterial supply of the motor tracts of the spinal cord from T8 to the conus medullaris? A. Medial spinal artery B. Median sacral artery C. Artery of Adamkiewicz D. Posterior spinal artery E. Lateral spinal artery
C. Artery of Adamkiewicz Discussion: The artery of Adamkiewicz originates from the left lower intercostal vessels from T8 to L2. It is located on the left side in 80% of cases and provides the main arterial supply for the motor tracts of the spinal cord from T8 to the conus. The other options do not supply the motor tracts because of their anatomical course. The lateral spinal artery supplies the posterior and lateral aspects of the spinal cords. The posterior spinal arteries supply the posterior one third of the spinal cord. The median sacral artery supplies the sacral canal. There is no medial spinal artery.
75
What is the most effective treatment in secondary stroke prevention in adults with Moyamoya disease? A. Dual antiplatelet therapy B. Anticoagulation C. Aspirin alone D. Direct revascularization E. Indirect revascularization
D. Direct revascularization Discussion: Direct revascularization, STA-MCA bypass, is the best treatment for secondary stroke prevention in adult patients with Moyamoya disease. Indirect revascularization (burr holes and synangiosis) works very well in pediatric patients but are less effective in adults. Medical therapy, whether that be antiplatelet or anticoagulation, is generally considered less effective than surgical therapy
76
A 54-year old female presented with recurrent headaches. The patient was found to have an arteriovenous malformation with a 4 cm diameter centered in the primary motor cortex. On angiography, the arteriovenous malformation was found to have deep draining veins. What is the Spetzler-Martin grade? A. Grade 2 B. Grade 5 C. Grade 4 D. Grade 1 E. Grade 3
C. Grade 4 Discussion: According to the Spetzler-Martin grading system, arteriovenous malformations are graded on the basis of size, pattern of venous drainage, and neurological eloquence of adjacent brain. Grade I malformations are small, superficial, and located in non-eloquent cortex. Grade V lesions are large, deep, and situated in neurologically critical areas.
77
Which one of the following is an absolute contraindication for administration of intravenous tissue plasminogen activator (tPA) for acute cerebral infarction? A. Abdominal surgery six weeks prior to stroke onset. B. Recent lumbar puncture C. Platelet count of 80,000. D. History of aneurysm clipping six months prior to stroke onset. E. Warfarin use with INR 1.3.
C. Platelet count of 80,000. Discussion: Absolute contraindications to intravenous tPA for acute cerebral infarction are based on safety guidelines as prescribed by the FDA for alteplase. Among these are recent surgery/trauma (less than 15 days), recent intracranial or spinal surgery, head trauma, or stroke (less than 3 months), seizure at onset of stroke symptoms, oral anticoagulant use with INR > 1.7, and a platelet count < 100,000. Relative contraindications to tPA include minor or rapidly improving symptoms, major surgery or serious non-head trauma in the previous 14 days, history of GI or urinary tract hemorrhage within 21 days, recent arterial puncture at noncompressible site, recent lumbar puncture, post myocardial infarction pericarditis, and pregnancy.
78
What structure is indicated by the arrow on this lateral projection angiogram? A. Internal cerebral vein B. Basal vein of Rosenthal C. Inferior saggital sinus D. Vein of Labbe E. Thalamostriate vein
D. Vein of Labbe Discussion: This is a large venous structure that appears to be in the vicinity of the deep venous system on this lateral projection. However, following its course, one can see that it leads to the transverse sinus. Hence, this is the Vein of Labbe. In addition, one can identify the following veins: thalamostriate vein, internal cerebral vein, basal vein of Rosenthal, vein of Galen, and straight sinus, none of which are connected to the indicated vein. The other answers name structures of the deep venous system while the indicated vein is a superficial structure.
79
What are the standard radiographic criteria for ventriculomegaly in order to diagnose hydrocephalus? A. Frontal horns are >50% of brain width B. Frontal horns are >30% of the brain width C. Frontal horns are >20% of the brain width D. Temporal horn's width >1 mm E. Temporal horn's width >0.5 mm
A. Frontal horns are >50% of brain width Discussion: Several radiographic criteria are found in hydrocephalus. The inner table shown beaten copper cranium, the sella shows erosion, the frontal horn’s percent of brain width is > 50%, the temporal horns’ width is more than 2 mm, the Anteroposterior (AP) view shows disproportion of the ventricle size and cortical sulci, the third ventricle on AP view shows bowing bilaterally and the third ventricle on lateral view shows bowing down into sella. The brain shows transependymal edema, the corpus callosum is thin and shows stretching and upward bowing.
80
After ischemic stroke, which medication is recommended for stroke prevention? A. Rivaroxaban B. Dabigatran C. Aspirin D. Unfractionated Heparin E. Warfarin
C. Aspirin Discussion: According to the latest guidelines for acute ischemic strokes, there is Class I-A level evidence that oral administration of aspirin within 24 to 48 hours after stroke onset is recommended for most patients. Currently available data demonstrate a small but statistically significant decline in mortality and unfavorable outcomes with the administration of aspirin within 48 hours after stroke. It appears that the primary effects of aspirin are attributable to a reduction in early recurrent stroke. Data regarding the utility of other antiplatelet agents, including clopidogrel alone or in combination with aspirin, for the treatment of acute ischemic stroke are limited. In addition, data on the safety of antiplatelet agents when given within 24 hours of intravenous fibrinolysis are lacking.
81
A pericallosal aneurysm would be found at which location on the lateral projection angiogram shown? A. C B. A C. E D. B E. D
D. B Discussion: The correct answer is B. Pericallosal aneurysms are typically found at the point where the anterior cerebral artery (ACA) bifurcates into the pericallosal artery and the callosomarginal artery, the two main branches of the main trunk of the ACA. This birfucation also marks the transition from the A2 to A3 segment of the ACA. (A) Indicates the distal callosomarginal artery. (C) indicates branches of the middle cerebral artery. (D) indicates the supraclinoid internal carotid artery, which aneurysms of the posterior communicating artery and anterior choroidal artery can be found. (E) indicates the origin of the ophthalmic artery, where carotid-ophthalmic and other paraclinoid carotid aneurysms can be found.
82
Aside from bilateral ICA occlusions, what is a typical angiographic finding in moyamoya disease? A. Duplication of the M1 segment of the middle cerebral artery B. Fetal posterior cerebral artery C. Persistent fetal trigeminal artery D. Diffuse hypertrophy of the lenticulostriate arteries E. Bilateral giant internal carotid artery aneurysms
D. Diffuse hypertrophy of the lenticulostriate arteries Discussion: Hypertrophy of the lenticulostriate arteries occurs in response to progressive occlusion of the terminal ICA, and produces the radiographic “puff of smoke” for which moyamoya is named. These arteries may develop small aneurysmal dilitations which can spontaneously rupture. However, giant ICA aneurysms are not associated with the disease. Duplication of the M1 is a variant of normal, and is not known to correlate with a pathologic state. The persistent fetal trigeminal artery is one of the more rare variations associated with moyamoya, along with fenestrated basilar systems, but these are not routinely associated with the condition. Fetal PCA too is not routinely associated with the condition.
83
A 60 year old male was found to have a 10 mm internal carotid artery aneurysm. The patient underwent surgical clipping of the aneurysm after which he experienced right hemiplegia, hemihypesthesia and homonymous hemianopsia. Which artery was most likely occluded? A. Anterior choroidal artery B. Anterior communicating artery C. Left A1 D. Left middle cerebral artery E. Recurrent artery of Heubner
A. Anterior choroidal artery Discussion: Occlusion of the anterior choroidal artery can lead to the complete triad of contralateral hemiplegia, hemihypesthesia and homonymous hemianopsia. Incomplete forms of this triad are common. The most common clinical sign is hemiparesis. Hemisensory loss is usually transient but may be severe at onset. Homonymous upper-quadrantanopsia, hemianopsia, or upper- and lower-quadrant sector anopsia can be present. Infarction in the recurrent artery of Heubner causes expressive aphasia and mild hemiparesis. Occlusion of the anterior communicating artery leads to contralateral weakness and sensory loss involving primarily the lower limbs, apraxia, anosmia and possible urinary incontinence. Occlusion of the middle cerebral artery leads to hemiparesis, hemisensory loss, hemineglect, aphasia, homonymous hemianopsia and Gerstmann syndrome with dominant parietal lobe infarct. Anterior cerebral artery occlusion can lead to contralateral hemiparesis and hemisensory loss with lower limb predominance, extrapyramidal syndromes, gait apraxia, abulia, incontinence, akinetic mutism and depression.
84
The North American Symptomatic Carotid Endarterectomy Trial (NASCET) reported the surgical results in patients who underwent carotid endarterectomy (CEA). According to the trial, what is the rate of perioperative permanent disabling stroke and death related to CEA? A. 5-10% B. 10-15% C. 20-25% D. <5% E. 15-20%
D. <5% Discussion: According to the North American Symptomatic Carotid Endarterectomy Trial (NASCET), the overall rate of perioperative stroke and death was 6.5%, but the rate of permanently disabling stroke and death was 2%. Other surgical complications were not clinically important. The study concluded that the carotid endarterectomy is a durable procedure.
85
A 76-year-old female with a history of diabetes mellitus and hypertension has asymptomatic atrial fibrillation. What medical management paradigm would best reduce her future risk of stroke? A. Aspirin and clopidogrel B. Warfarin and clopidogrel C. Clopidogrel D. Aspirin E. Warfarin
E. Warfarin Discussion: For patients with CHADS2 score >=2, warfarin therapy is significantly protective for stroke. Antiplatelet therapy is effective in protecting against stroke if the patient has a CHADS score of 0 or 1. This patient’s CHADS2 score is 3 and thus warfarin would be the correct choice.
86
How does the amount of radiation differ when standing 4 feet away from a radiation source as opposed to 1 foot? A. 1/8 B. 1/12 C. 1/64 D. 1/16 E. 1/4
D. 1/16 Discussion: The inverse square law states that the amount of radiation is proportional to the inverse square of the distance away from the radiation source. Therefore, the amount of radiation 4 feet away is 16 times (42) less than 1 foot away.
87
Which finding on clinical exam can distinguish a third cranial neuropathy that is caused by an aneurysm from that caused by diabetic neuropathy? A. pupillary dilation B. double vision C. pain D. ptosis E. miosis
A. pupillary dilation Discussion: Third cranial neuropathy can be induced by compression from an aneurysm. The most common aneurysm to do this is a posterior communicating aneurysm. Diabetic neuropathy causes a pupil sparing third nerve neuropathy. As shown in the diagram, control of ocular muscles is via fibers that are central within the nerve. Miosis is controlled by parasympathetic fibers which are external in the nerve. A posterior communicating artery aneurysm will affect the external fibers fiompression but can later also affect ocular muscles through ischemia and further compression. Diabetic neuropathy on the other hand will affect the deep fibers of the nerve early because the vessel that gets affected by glycosylation supplies the output to the ocular muscles preferentially.
88
What factor significantly impacts the rerupture risk of a previously treated ruptured intracranial aneurysm? A. Hypertension B. Age at initial rupture C. Degree of aneurysm occlusion D. Fisher score E. Gender
C. Degree of aneurysm occlusion Discussion: Raymond–Roy Occlusion Classification (RROC; also known as the Montreal Scale, Modified Montreal Scale, or the Raymond Montreal Scale) is a widely accepted system for evaluating aneurysm occlusion class. In this scheme, Class I is defined as complete obliteration, Class II as residual neck, and Class III as residual aneurysm. It has been shown that Class III aneurysms have a higher propensity to remain incompletely occluded and potentially rebleed. According to the CARAT study, there was a strong association between the degree of occlusion of a previously treated ruptured intracranial aneurysm and the rerupture risk of the aneurysm. Cumulative Risk: 1.1% for complete occlusion, 2.9% for 91% to 99% occlusion, 5.9% for 70% to 90% occlusion, 17.6% for < 70%; P<0.0001). Labile hypertension is a general risk factor for aneurysm rupture. Age, gender and Fisher score do not pose risk for aneurysm rupture.
89
What is the most common location for a hypertensive hemorrhage? A. Amygdala B. Cerebellum C. Basal Ganglia D. Pons E. Thalamus
C. Basal Ganglia Discussion: Explanation: The putamen is the most common site for hypertensive intracerebral hemorrhage. It is characterized by contralateral hemiparesis that can progress to hemiplegia, coma and death. Rare findings include papilledema and subhyaloid preretinal hemorrhage. By decreasing order of frequency, the common sites for hemorrhage are basal ganglia (50%), thalamus, pons (10-15%), Cerebellum (10%).
90
According to the North American Symptomatic Carotid Endarterectomy Trial (NASCET), symptomatic patients with 70-99% carotid stenosis are best treated by: A. Warfarin B. Carotid endarterectomy C. Aspirin and clopidogrel D. Observation E. Aspirin
B. Carotid endarterectomy Discussion: Endarterectomy in patients with symptomatic moderate carotid stenosis of 50 to 69 percent yielded only a moderate reduction in the risk of stroke. Decisions about treatment for patients in this category must take into account recognized risk factors, and exceptional surgical skill is obligatory if carotid endarterectomy is to be performed. Patients with stenosis of less than 50 percent did not benefit from surgery. Patients with severe stenosis (≥70 percent) had a durable benefit from endarterectomy at eight years of follow-up compared to aspirin alone. Warfarin and clopidogrel were not assessed in this trial
91
A patient presents with subarachnoid hemorrhage from intracranial vertebral artery dissection. What is the definitive treatment? A. Intravenous heparin B. Microsurgical or endovascular intervention C. Aminocaproic acid D. Aspirin E. Aspirin and clopidogrel
B. Microsurgical or endovascular intervention Discussion: The correct answer is: microsurgical or endovascular intervention. Patients with a subarachnoid hemorrhage following vertebral artery dissection should receive surgical or endovascular intervention. Surgical or endovascular treatment is indicated for a number of patients with arterial dissections including those who have persistent symptoms of ischemia despite adequate anticoagulation, symptomatic flow limitation, or intracranial hemorrhage. Patients with subarachnoid hemorrhage from dissections or dissecting aneurysms are at high risk for rehemorrhage and require intervention.
92
A 40 year-old woman presents with acute headache, mild right arm weakness and a CT scan showing a small hemorrhage in the left frontoparietal region. Cerebral angiogram shows a 4 cm compact, superficial AVM with superficial venous drainage and deep perforating arteries. Which of the following factors is most associated with neurological outcomes after surgery? A. Superficial venous drainage B. Superficial location C. Ruptured Presentation D Diffuse architecture E. Deep location
D Diffuse architecture Discussion: The Lawton-Young supplementary AVM score improves the predictive accuracy of the Spetzler-Martin grade and is now widely used in the evaluation of surgical risk for patients with AVMs. Factors predictive of increased surgical risk in the Spetzler-Martin score include size, deep venous drainage, and eloquence. In the Lawton-Young scale, factors predicting increased surgical risk include older age, unruptured presentation, and diffuse nidus. Deep perforating arterial supply was not significantly associated with worsened postoperative outcomes in Lawton's score, but it was one of the factors that they examined. Deep location is not one of the factors associated with worse outcomes in S-M or L-Y grades.
93
A 44 year old man with a history of blunt head trauma 4 days earlier presents with a progressively swollen and red right eye, headache, and double vision. What is the most likely cause? A. Direct, high flow lesion between ICA and cavernous sinus B. Indirect, low flow lesion between meningeal ICA branches and cavernous sinus C. Cervical internal carotid dissection D. Indirect, low flow lesion between both ICA and ECA meningeal branches and the cavernous sinus E. Vertebral artery dissection
A. Direct, high flow lesion between ICA and cavernous sinus Discussion: The most common lesion after direct trauma is a type A carotid-cavernous fistula with a direct connection between the cavernous carotid artery and the cavernous sinus. A patient may develop a lesion even if the initial CT head is negative for fracture. The other choices are indirect lesions feeding from the ICA meningeal branches (type B), the ECA meningeal branches (type C), or both ICA/ECA meningeal branches (type D). Carotid dissection typically presents with neck pain, Horner’s syndrome, and hemispheric stroke symptoms. Vertebral dissection typically presents with neck pain and neurological deficits attributable to the posterior circulation, but would not include orbital findings.
94
A 38 year-old presents with a one month history of difficulty swallowing, mild hoarseness and right facial numbness and tingling. The imaging studies are shown below. What is the most likely diagnosis? A. Medulloblastoma B. PICA aneurysm C. Arteriovenous malformation D. Capillary hemangioma E. Cavernous angioma
E. Cavernous angioma Discussion: MRI is the diagnostic test of choice for cavernous angioma of the brain. The lesion appears with characteristic “popcorn” or "berry" appearance with a rim of signal loss due to hemosiderin, which demonstrates prominent blooming on susceptibility weighted sequences. Gradient Echo or T2* sequences can delineate these lesions better than T1 or T2 weighted images. Susceptibility imaging (SWI) may have sensitivity that is superior to GRE in detecting small cavernomas.. SWI is also highly sensitive in detecting calcification as compared to T1 and T2 images.
95
What neurological deficit may result from embolization of the artery with liquid embolics indicated on this angiogram? A. Unilateral facial palsy B. Blindess C. Contralateral hemiparesis D. Unilateral tongue atrophy E. Loss of smell
D. Unilateral tongue atrophy Discussion: The indicated artery is the lingual artery, a named branch of the external carotid artery. Unlike other branches, such as the facial artery and the occipital artery, the lingual artery does not have robust collaterals with the contralateral side and embolization of this branch, particularly with an agent with deep penetrance such as a liquid agent, will often lead to unilateral atrophy of the tongue. The lingual artery is typically the second anterior branch of the external carotid artery, the first being the superior thyroid artery and the third being the facial artery. The ascending pharyngeal artery typically originates off the back wall of the external carotid artery between the superior thyroid artery and the lingual artery. Hemiparesis, blindness, and loss of smell are not typically complications associated with branches of the external carotid artery. Embolization of the ascending pharyngeal artery can occlude perforators supplying the lower cranial nerves, specifically those around the jugular foramen. Occasionally, the seventh and eighth nerves can be affected by this as well.
96
In Yasargil’s classification, a type 4 vein of Galen malformation (VGA) is: A. A VGA associated with a cavernoma B. A direct fistulous communication with the vein of Galen C. A parenchymal arteriovenous malformations (AVMs) which drains into the vein of Galen D. A VGA associated with an aneurysm E. A VGA associated with hydrocephalus
C. A parenchymal arteriovenous malformations (AVMs) which drains into the vein of Galen Discussion: The two most widely used classification systems for vein of Galen malformations have been provided by Yasargil and Lasjaunias. Yasargil classified VOGMs into four categories. Type 1, 2 and 3 lesions in Yasargil's classification involve a direct fistulous communication with the vein of Galen. There is no other proximal nidus. Type 4 lesions represent parenchymal arteriovenous malformations (AVMs), which drain into the vein of Galen. Hydrocephalus is not a component of the VOG grade.
97
According to the International Study of Unruptured Intracranial Aneurysms (ISUIA), what is the risk of rupture of a cavernous carotid artery aneurysm (< 7 mm) in a patient with a history of previous aneurysmal subarachnoid hemorrhage? A. 0% B. 3% C. 2% D. 4% E. 1%
A. 0% Discussion: According to the International Study of Unruptured Intracranial Aneurysms (ISUIA), the 5 year risk of rupture of cavernous carotid artery aneurysms (< 7 mm in size) in patients with or without a history of aneurysmal rupture is zero. (Group 1: patients without subarachnoid hemorrhage from a separate aneurysm. Group 2: patients with subarachnoid hemorrhage from a separate aneurysm).
98
According to Suzuki and Takaku classification of the angiographic appearance of moyamoya disease, what imaging findings are asssociated with stage 2 disease? A. Near complete disappearance of the major cerebral arteries with the cerebral hemispheres receiving blood from the abnormal extracranial-intracranial anastomoses. B. Diminishing moyamoya vessels with progression of extracranial circulation C. Stenosis of the carotid artery at the suprasellar portion with no moyamoya vessels D. Moyamoya vessels begin to develop at the base of the brain
D. Moyamoya vessels begin to develop at the base of the brain Discussion: The original classification of moyamoya disease by Suzuki and Takaku was intended to stratify patients according to the severity of the disease. Stage 1 Stenosis of the carotid artery at its suprasellar portion, usually bilateral Stage 2 Moyamoya vessels begin to develop at the base of the brain Stage 3 Moyamoya vessels are prominent as major trunks in the anterior circulation become severely occluded Stage 4 Posterior Cerebral Arteries occluded, moyamoya vessels begin to diminish and collateral pathways from extra cranial circulation develop Stage 5 Moyamoya vessels are diminishing and extra cranial circulation progress Stage 6 Moyamoya vessels and the major cerebral arteries completely disappear, the cerebral hemispheres receives blood through the abnormal extra cranial intracranial anastomosis
99
What is the vascular malformation demonstrated in this figure? A. Capillary telangiectasis B. Venous angioma C. Arteriovenous malformation D. Cavernous malformation E. Arteriovenous fistula
B. Venous angioma Discussion: Developmental venous anomalies, also known as venous angiomas, are a variation of normal venous drainage characterized by a tuft of medullary veins that converge into a large central trunk that drains either to the deep or superficial venous system. These lesions have a characteristic “caput medusa” or sunburst appearance on angiography. Arteriovenous malformations are a tangle of abnormal blood vessels and angiographically appear as a typical nidus and early draining vein(s). Arteriovenous fistulas can be pial or dural and angiographically consist of an abnormal connection between artery and an early draining vein. Cavernous malformations consist of clusters of thin-walled blood vessels and are typically angiographically occult, meaning they do not appear on angiography. Capillary telengiectasias are comprised of dilated capillaries and are interspersed with normal brain parenchyma, and are angiographically occult, meaning they do not appear on angiography.
100
What is the mechanism of action of clopidogrel (Plavix)? A. Inhibition of plasminogen B. Activation of antithrombin III C. Inhibition of the adenosine diphosphate (ADP) chemoreceptor, P2Y D. Direct inhibition of factor Xa E. Direct inhibition of thrombin
C. Inhibition of the adenosine diphosphate (ADP) chemoreceptor, P2Y Discussion: In patients with unruptured aneurysms, coiling is often performed with an initial 100 u/Kg of Heparin bolus and maintenance of activated clotting time of 2 times the patient's baseline intraoperatively. When the use of a stent is anticipated, patients are often pretreated with 81 mg of aspirin and 75mg of clopidogrel 10 days prior to the procedure. Patients treated often receive platelet function tests on a routine basis to ascertain that the level of platelet inhibition is adequate. Reponse to plavix is tested by the P2Y12 assay. For stent-assisted procedures performed in the setting of a subarachnoid hemorrhage, patients are often loaded with 600 mg of clopidogrel intraprocedurally and 50 U/Kg of Heparin bolus following deployment of the first coil. Clopidogrel inhibits P2Y, an adenosine diphosphate (ADP) chemoreceptor. This blocks activation of the glycoprotein IIb/IIIa pathway on platelet cell membranes ad inhibits aggregation. Heparin activates antithrombin III (AT). The activated AT then inactivates thrombin and other proteases involved in blood clotting, most notably factor Xa. Low-molecular-weight heparins and fondaparinux target anti-factor Xa activity rather than anti-thrombin (IIa). With LMWH and fondaparinux, there is a reduced risk of osteoporosis and heparin-induced thrombocytopenia (HIT), although in aneurysm patients receiving heparin at the time of angiogram, the incidence HIT may not be decreased when placed on LMWH for venous prophylaxis. Danaparoid sodium works by inhibiting factor Xa. Danaparoid, a mixture of heparan sulfate, dermatan sulfate, and chondroitin sulfate, can be used as an anticoagulant in patients that have developed HIT. Because danaparoid does not contain heparin or heparin fragments, cross-reactivity of danaparoid with heparin-induced antibodies Dabigatran is a direct thrombin inhibitor which is undergoing trials for use in patients with atrial fibrillation, pulmonary embolism, deep venous thrombosis, and venous thromboembolism prophylaxis. Unfortunately, there is no current approved means of reversal, making neurosurgical management of patients on dabigatran more complex. Aminocaproic acid is an inhibitor of plasminogen. Plasminogen activates plasmin to break down thrombin and activate fibrinolysis. Thus minocaproic acid is an antifibrinolytic that helps stabilize clotting. There is some evidence that it may inhibit aneurysmal rebleeding when used acutely for short periods in patients with subarachnoid hemorrhage.
101
According to randomized control trials of hemicraniectomy for malignant MCA infarction, what is the time frame for the performance of hemicraniectomy associated with reduced mortality? A. 24 hours B. 6 hours C. 3 hours D. 48 hours E. 12 hours
D. 48 hours Discussion: Metaanalysis from 3 randomized control trials (DECIMAL, DESTINY, HAMLET) found that hemicraniectomy performed within 48 hours decreases mortality and increases the number of patients with favorable functional outcomes. Medical treatment alone does not provide the best outcomes and is associated with higher risk of morbidity and mortality. Intra-arterial tPA would not decrease the risk of mortality and can be administered up to 6 hours after the onset of the stroke. There is no current evidence that EVD placement and mechanical thrombectomy would decrease the risk of mortality in malignant CA strokes. The Middle cerebral artery territory infarction with associated malignant edema is best treated with decompressive surgery. Decompressive surgery for malignant cerebral edema is effective and can be life saving. Factors such as age and patient/family valuation of outcomes may affect the decision regarding the surgery. When edema produces an increase in the intracranial pressure, each of hyperventilation, hypertonic saline osmotic diuretics and intraventricular drainage of cerebrospinal fluid can be performed. Early reperfusion of large necrotic tissue can precipitate and accelerate edema to a critical level in the first 24 hours resulting in what is called as “malignant edema”. Decompressive craniectomy is the therapeutic option of choice in malignant edema.
102
A 72-year-old female presents with an incidentally discovered lesion as shown in the figure. What is her estimated 5-year risk of hemorrhage or new neurological deficit related to this lesion? A. 41-50% B. 11-20% C. 21-30% D. 31-40% E. 0-10%
E. 0-10% Discussion: The natural history of cavernous malformations varies depending on location (brainstem versus other) and initial presentation (intracranial hemorrhage or focal neurological deficit versus other). Horne et al demonstrate that brainstem location and hemorrhagic lesions portend a more aggressive clinical course, with an estimated 5-year recurrent hemorrhage risk of 30.8%, with a 50.7% 5-year risk of developing either a recurrent hemorrhage or a new neurological deficit not related to hemorrhage. Meanwhile, asymptomatic supratentorial cavernomas have an approximate 3.7% 5-year risk of developing either a recurrent hemorrhage or a new neurological deficit not related to hemorrhage.
103
What is the appropriate sequence of steps during AVM microsurgery ? A. nidus resection - ligation of the draining vein(s) – coagluaiton of feeding arteries B. coagulation of feeding arteries – nidus resection – ligation of the draining vein(s) C. ligation of the draining vein(s) – nidus resection – coagluaiton of feeding arteries D. coagulation of feeding arteries – ligation of the draining vein(s) – nidus resection E. ligation of the draining vein(s) - coagulation of feeding artery - nidus resection
B. coagulation of feeding arteries – nidus resection – ligation of the draining vein(s) Discussion: As described by Yaşargil, AVM resection follows the sequence of coagulating the feeding arteries, separating the nidus from brain tissue while leaving the draining vein ligation until the end. It is important to note that complete hemostasis of the cavity should be achieved before closing due to the loss autoregulation around the cavity and the propensity of the surgical cavity floor to re-erupt. Ligation of the draining veins before coagulation of arteries and nidus resection could increase the perfusion pressure in the AVM and increase the risk of rupture.
104
What structure does the arrow point to in the figure? A. Internal jugular vein B. Superior ophthalmic vein C. Superior petrosal sinus D. Cavernous sinus E. Inferior petrosal sinus
B. Superior ophthalmic vein Discussion: The image shows a classic CCF with an arterialized superior ophthalmic vein. The superior ophthalmic vein usually drains towards the cavernous sinus which in turns drains towards the superior and inferior petrosal sinus. The sigmoid sinus drains to the internal jugular vein.
105
A 30-year old female presented with left third nerve palsy. The patient was found to have a posterior communicating artery aneurysm and was scheduled for surgery. Which anatomic variation could result in occipital lobe infarction if the posterior communicating artery is occluded? A. Multiple aneurysms B. Vertebral artery hypoplasia C. Persistent trigeminal artery D. Fetal posterior cerebral artery E. Aneurysmal size <15 mm
D. Fetal posterior cerebral artery Discussion: A fetal posterior cerebral artery circulation, implies that the posterior cerebral artery is only supplied through the posterior communicating artery. Occluding a fetal posterior communicating artery could result in occipital lobe infarction. A large aneurysm alone would not make posterior communicating artery occlusion necessarily problematic although it is always preferable to preserve parent arteries. Likewise the presence of multiple aneurysms is not independently associated with posterior communicating artery dominance. Vertebral artery hypoplasia could be associated with a fetal circulation particularly if bilateral but not necessarily. Persistent trigeminal artery is a congenital anastomosis of the internal carotid artery to the basilar artery proximal to the PCA.
106
To what structure does the arrow point? A. Persistent otic artery B. McConnel's Capsular artery C. Caroticotympanic artery D. Vidian artery E. Persistent stapedial artery
D. Vidian artery Discussion: The arrow points to the Vidian artery which arises from the petrous segment of the internal carotid artery (ICA). The Vidian branch courses forward to enter the Vidian canal with the vidian nerve and exits to meet the internal maxillary artery. This branch may be enlarged as it is in this case as it is supplying juvenile angiofibroma. The mandibular branch does not enter the Vidian canal but travels anterior and inferior to supply the pterygoid muscles and posterior pharyngeal wall. The persistent otic artery may fail to involute and provide a connection from the ICA to the basilar artery. The persistent stapedial artery may also fail to involute and courses from the vertical ICA petrous segment to the middle meningeal artery. The caroticotympanic artery rises near the genu to pass superiorly to supply the middle and inner ear. Aberrant course can cause a retrotympanic pulsatile mass. McConnel’s capsular artery arises from the intracavernous ICA and supplies the anterior and inferior pituitary gland.
107
During clipping of an anterior communicating artery aneurysm, an artery arising at the junction of the A1 and A2 segments was inadvertently occluded. This resulted in expressive aphasia and mild hemiparesis. The artery was most likely the: A. Right A2 B. Recurrent artery of Heubner C. Orbitofrontal artery D. Frontopolar artery E. Anterior choroidal artery
B. Recurrent artery of Heubner Discussion: The recurrent artery of Heubner arises from the junction of the anterior cerebral artery and anterior communicating artery in 62.3%, proximal A2 in 23.3%, and A1 in 14.3%. The artery courses back on the anterior cerebral artery and accompanies it and the middle cerebral artery before entering the anterior perforated substance. The recurrent artery of Heubner vascularizes the anteromedial section of the curate nucleus and the anterioinferior section of the internal capsule as well as parts of the putamen and septal nuclei. Occlusion of the recurrent artery of Heubner leads to expressive aphasia and hemiparesis affecting the lower extremities more than the upper extremities. Occlusion of the anterior choroidal artery causes a triad of hemiparesis, homonymous hemianopsia and hemihypesthesia. The orbitofrontal and frontopolar arteries supply the frontal lobes mainly and thus would not cause such symptoms. A2 branch occlusion might lead to contralateral hemiparesis and hemisensory loss with lower limb predominance, extrapyramidal syndromes, gait apraxia, abulia, incontinence, akinetic mutism and depression.
108
Which of the following cerebrovascular pathologies exhibits early venous drainage on an angiogram? A. Capillary telangiectasia B. Cavernous angioma C. Amyloid angiopathy D. Venous angioma E. Arteriovenous Malformation
E. Arteriovenous Malformation Discussion: Early venous drainage can be seen in a number of cerebral lesions including stroke, certain tumors and arteriovenous malformations. The presence of early venous drainage can help narrow the differential diagnosis. Typical lesions that tend to manifest this feature include arteriovenous malformations, vein of Galen malformations and arteriovenous fistulas. Venous angiomas , cavernomas , capillary telangiectasias and amyloid angiopathy do not exhibit early venous drainage.
109
Five days after the rupture of a saccular aneurysm in a 30 year old female, the patient experiences altered level of consciosness. Transcranial Doppler shows a Lindegaard (MCA-ICA) ratio of 5. What is the most likely diagnosis? A. Severe Vasospasm B. Normal Finding C. Mild vasospasm D. Hyperemia E. Rebleed
C. Mild vasospasm Discussion: Transcranial Doppler is a noninvasive method to diagnose cerebral vasospasm. An increase of arterial flow may only suggest the diagnosis. An MCA:ICA (Lindegaard) ratio can distinguish hyperemia (which increase blood flow velocity in both the ICA and MCA) from vasospasm. A ratio less than 3 is considered normal, a ratio between 3 and 6 is specific for mild vasospasm and a ratio higher than 6 is specific for severe vasospasm. Although the Lindegaard ratio is very specific for the diagnosis of cerebral vasospasm, it is only 60% sensitive.
110
A 39-year-old female presents with subarachnoid hemorrhage. Her angiogram is shown. What is the most likely diagnosis? A. Supraclinoid ICA dissection B. Infundibulum C. Blister aneurysm D. Fusiform aneurysm E. Saccular Aneurysm
B. Infundibulum Discussion: An infundibulum is most commonly found in the posterior communicating artery, is triangular in shape, is less than 3mm and the vessel origin is usually found on the apex. It is identified in 7-13% of otherwise normal arteriograms and can be mistaken for a saccular aneurysm. Although they may bleed, there is significantly less risk of rupture than a saccular intracranial aneurysm. The conical shape, the size of the lesion and its location are diagnostic for an infundibulum of the posterior communicating artery.
111
What is the most appropriate management of a fusiform 4-mm M3 unruptured mycotic aneurysm which is found in a patient with bacteremia and fever? A. Antibiotics for 4 - 6 weeks B. Vascular bypass surgery C. Direct surgical clipping D. Endovascular coiling E. Observation
A. Antibiotics for 4 - 6 weeks Discussion: Most mycotic aneurysm cases are treated acutely with antibiotics for 4-6 weeks. Serial angiography can help document effectiveness of medical therapy. Indications for clipping, bypass or endovascular treatment may include subarachnoid hemorrhage, or increase in aneurysmal size despite medical therapy. Observation without antibiotics likely increase the risk of rupture of the aneurysm.
112
Anterograde blood flowing in the indicated vascular structure on this lateral projection cerebral angiogram drains into which structure? A. Vein of galen B. Inferior sagittal sinus C. Cavernous sinus D. Inferior petrosal sinus E. Superior petrosal sinus
A. Vein of galen Discussion: The indicated vascular structure is the internal cerebral vein, which runs in the roof of the third ventricle between the two leaves of the velum interpositum. Blood from here drains posteriorly into the Vein of Galen and subsequently the straight sinus. The inferior sagittal sinus is a dural sinus that joins with the vein of Galen to form the straight sinus. The superior and inferior petrosal sinuses are based on the temporal bone and typically receive blood from the cavernous sinus and lateral convexity veins and typically do not receive drainage from the deep venous system. The cavernous sinus is a skull base structure that also does not typically receive blood from the deep venous system
113
A 50 year old male is diagnosed with an acute ischemic stroke. He is a heavy smoker and admits occasional use of marijuana and oxycodone. He has a past medical history of deep vein thrombosis and varicose veins. What risk factor most likely contributed to his stroke? A. Varicose veins B. Deep venous thrombosis C. Opioids D. Smoking E. Marijuana
D. Smoking Discussion: Smoking is a well-established risk factor from stroke with a relative risk of 1.5-2.0. Marijuana, opioids and varicose veins alone do not constitute risk factors for stroke. Deep venous thrombosis is a rare cause for stroke and occurs when an embolus travels from the leg veins to a heart with atrial septal defects (paradoxical embolus).
114
What is the Fisher score in a patient with a 2 mm thick subarachnoid hemorrhage with no intraventricular or parenchymal extension? A. II B. V C. I D. III E. IV
D. III Discussion: The Fisher classification of Subarachnoid hemorrhage (SAH) was designed as a predictor of vasospasm risk in SAH patients. The table below summarizes the different fisher grades. There is no Fisher score V. Score Appearance of hemorrhage I None evident II Less than 1 mm thick III More than 1 mm thick IV Diffuse or none with intraventricular hemorrhage or parenchymal extension
115
Which factor has been strongly associated with increased hemorrhage rate after radiosurgery of an arteriovenous malformation (AVM)? A. Superficial AVM location B. Margin dose C. Number of prior hemorrhages D. Target volume E. Number of feeding arteries to the AVM
C. Number of prior hemorrhages Discussion: In their series of 47 patients with AVMs, Kano et al found that the number of prior hemorrhages was significantly associated with a higher rate of hemorrhage after stereotactic radiosurgery. Other listed options did not significantly affect the risk of hemorrhage. Furthermore, the presence of coexisting aneurysms proximal to the AVM correlates significantly with higher hemorrhage risks.
116
Twelve months after undergoing Gamma Knife radiosurgery to treat an AVM, a patient experiences headaches and a seizure. What is the underlying cause of the new onset of symptoms and edema shown on the attached CT and MRI? A. Hemorrhage from the nidus B. Radiation necrosis C. Secondary tumor D. Peri-lesional cyst E. Acute thrombosis of the draining vein
E. Acute thrombosis of the draining vein Discussion: The correct answer is acute thrombosis of the draining vein. Radiation–induced imaging changes and/or brain edema is not an unusual mid-term complication following radiosurgery for arteriovenous malformations. Multiple causes have been proposed to explain the imaging change, including: radiation injury of glial cells, endothelial cell damage followed by breakdown of the blood brain barrier, excessive generation of free radicals, and early thrombosis/occlusion of the draining veins. In this case, the CT scan demonstrates a high-density thrombus within the draining vein, and appears to be the cause of venous hypertension and brain edema. Following radiosurgery, collateral pathways for venous drainage develop for the the vasculature surrounding the nidus. Uncommonly, an acute/subacute thrombosis within the draining vein occurs before obliteration of the nidus and can cause venous hypertension and/or cerebral hemorrhage. Rupture of arteriovenous malformations during the latency period before nidus obliteration is a well-known risk for patients following radiosurgery. Additionally, cyst formation and secondary tumors are rare, but well documented long-term complications of radiosurgery for arteriovenous malformations. The imaging findings of this case are not consistent with a hemorrhage, cyst formation, or secondary tumor.
117
A 40 year old patient presents with headache and aneurysmal subarachnoid hemorrhage. On physical exam she was awake, alert and nonfocal with the exception of a right third nerve palsy and evidence of nuchal rigidity. What is the Hunt and Hess grade for this patient? A. 4 B. 3 C. 2 D. 1 E. 5
C. 2 Discussion: The following is the Hunt and Hess grade classification for SAH: Grade Description 1 Asymptomatic or mild headache and slight nuchal rigidity 2 Cranial nerve palsy on exam, moderate to severe headache, nuchal rigidity 3 Mild focal deficit, lethargy, or confusion 4 Stupor, moderate to severe hemiparesis, early decerebrate rigidity 5 Deep coma, decerebrate rigidity, moribund appearance In the presence of a serious disease or vasospasm add 1 grade
118
Which calcium channel blocker has been demonstrated to improve outcome in ptients with aneurysmal subarachnoid hemorrhage? A. IV Felodipine B. Oral Felodipine C. IV diltiazem D. Oral verapamil E. Oral nimodipine
E. Oral nimodipine Discussion: The latest SAH guidelines endorse oral nimodipine for all patients with aneurysmal SAH. The agent has been shown to improve neurological outcomes but not cerebral vasospasm. The evidence behind this recommendation is shown mainly by 1 clinical trial published in NEJM and 1 meta-analysis of the Cochrane database. In the clinical trial, Allen et al. enrolled 125 neurologically intact patients within 96 hours of onset of SAH in a prospective double blinded randomized placebo controlled trial to determine whether treatment with CCB nimodipine would prevent or reduce the severity of ischemic neurologic deficits that would result from arterial spasm. The study showed that the placebo group had significantly more incidents of deficits from arterial spasm that were persistent or severe compared to the nimodipine group. In their meta-analysis, Dorhout et al examined 16 randomized controlled trials involving 3361 patients. The results showed calcium antagonists reduced the risk of poor outcome: the relative risk (RR) was 0.81 (95% confidence interval (CI) 0.72 to 0.92); the corresponding number of patients needed to treat was 19 (95% CI 1 to 51). For oral nimodipine alone the RR was 0.67 (95% CI 0.55 to 0.81), for other calcium antagonists or intravenous administration of nimodipine the results were not statistically significant. The value of other calcium antagonists, whether administered orally or intravenously, remains uncertain.
119
Which artery is indicated by the arrow in the figure? A. Anterior chorodial artery B. Bernasconi and Cassinari C. Opthalmic artery D. Posterior communicating artery E. Vidian Artery
B. Bernasconi and Cassinari Discussion: The arrow points to the artery of Bernasconi and Cassinari which arises from the meningohypophyseal trunk. The meningohypophyseal trunk arises from the distal genu of the cavernous carotid artery. The three major branches of the MHT are the artery of Bernasconi and Cassinari, inferior hypophseal artery and clival dural branches. The artery of Bernasconi and Cassinari may be enlarged in pathological conditions, for example, tentorial meningiomas. In the current case, an enlarged artery of Bernasconi and Cassinari feeds a dural arteriovenous fistula with isolated perimedullary spinal drainage. Ophthalmic artery is typically the first intradural branch of the carotid artery, followed by the posterior communicating artery and then the anterior choroidal artery . The Vidian artery arises from the petrous segment of the internal carotid artery.
120
A 65-year-old male with a history of hypertension and diabetes presents to the emergency room after experiencing severe headache. The Glasgow Coma Scale (GCS) is 14. On head CT scan, the patient was found to have a 2 cm acute cerebellar hematoma with no hydrocephalus. What is the most appropriate management? A. Medical management in an ICU setting B. Stereotactic aspiration and infusion of thrombolytic agents C. External ventricular drainage D. Posterior fossa craniectomy E. Surgical evacuation
A. Medical management in an ICU setting Discussion: Randomized trials of surgical management of spontaneous intracerebral hemorrhage have not included patients with cerebellar hemorrhage. The AHA/ASA guidelines suggest surgical evacuation for cerebellar clots greater than 3cm in diameter or those that are associated with brainstem compression or hydrocephalus. The evidence behind this recommendation is retrieved from a number of clinical series that have shown significantly better clinical outcomes in patients who undergo surgery compared to those who are managed medically. If the hemorrhage is less than 3 cm in diameter and there is no brainstem compression nor hydrocephalus, reasonable outcomes can be achieved without surgery. A 2 cm clot can be managed medically in a stable patient in an intensive care unit with close observation.
121
What is the mortality of acute basilar artery occlusion? A. 50% B. 15% C. 90% D. 30% E. 5%
C. 90% Discussion: The acute basilar artery occlusion remains a disease with an extremely high mortality that can reach 90%.
122
What is the most common location of cranial dural arteriovenous fistulas? A. Inferior sagital sinus B. Transverse sinus C. Posterior cavernous sinus D. Straight sinus E. Superior sagital sinus
B. Transverse sinus Discussion: Dural AVMs involve the dural venous sinuses. The most common location is the transverse sinus (63% of cases), with a slight left-sided predominance. Their epicenter is at the junction of the transverse and the sigmoid sinus. Dural AVMS do occur in the tentorium, posterior cavernous sinus, sagittal sinus, and in the anterior cranial fossa but less commonly than the transverse sinus.
123
A 50-year-old male patient with a history of atrial fibrillation presents with acute onset left hemiparesis and drowsiness for 24 hours. MRI of the brain is shown. The National Institute of Health Stroke Scale (NIHSS) score is 16. What treatment option has been shown to decrease mortality rates for such a patient? A. Medical management only B. External ventricular drainage C. Decompressive hemicraniectomy D. Mechanical thrombectomy E. Intra-arterial tPA
C. Decompressive hemicraniectomy Discussion: Metaanalysis from 3 randomized control trials (DECIMAL, DESTINY, HAMLET) found that hemicraniectomy performed within 48 hours decreases mortality and increases the number of patients with favorable functional outcomes. Medical treatment alone is associated with higher risk of morbidity and mortality. Intra-arterial tPA can be administered up to 6 hours after the onset of the stroke. This patient presented 24 hours after the symptoms occurred and is hence not a candidate for IA tPA. There is no current evidence that EVD placement and mechanical thrombectomy would decrease the risk of mortality in malignant MCA strokes.
124
A 24 year old man presents to the ER with an acute subdural hematoma (ASDH) after falling down the stairs. CT scan shows a hematoma of 13 mm thickness. What is the appropriate management? A. Medical management if Glascow Coma Scale (GCS) >11 B. Medical Management is GCS <10 C. Surgical Evacuation only if GCS>11 D. Surgical Evacuation only if GCS<10 E. Surgical evacuation regardless of the GCS
E. Surgical evacuation regardless of the GCS Discussion: Acute Subdural Hematoma (ASDH) with thickness > 10 mm or midline shift >5 mm on CT should be evacuated regardless of GCS. ASDH with thickness < 10 mm and MLS < 5 mm should undergo surgical evacuation if: GSC drops by >2 points from injury to admission, and/or the pupils are asymmetric or fixed and dilated, and/or ICP is > 20 mmHg.
125
What is the most likely appearance of a chronic subdural hematoma on MRI? A. Hypointense on both T1 and T2 B. Hyperintense on both T1 and T2 C. Isointense on both T1 and T2 D. Hypointense on T1 and hyperintense on T2 E. Hyperintense on T1 and hypointense on T2
B. Hyperintense on both T1 and T2 Discussion: Discussion: Typically, a chronic subdural hematoma is hyperintense on both T1 and T2 weighted images, but about 30% of them may be either isointense or hypointense on T1 weighted images. The T1 values of chronic subdural hematoma is significantly shorter than gray matter values and significantly longer than white matter values. The T2 values are significantly longer than both gray and white matter values. In the hyperacute phase, patients show isointense hematoma compared to the adjacent cortex. Classically in the acute phase, the hematoma has a crescent shape and starts increasing in hyperintensity (hypo and hyperintense mixed). In the subacute phase the hematoma becomes isointense to the cortex.
126
A 54-year old Hispanic male was incidentally found to have two cavernous malformations located in the left frontal and right temporal lobes on CT scan. What is the best study to rule out additional cavernous malformations? A. Diffusion weighted magnetic resonance imaging B. Contrast enhanced computed tomography C. Gradient echo magnetic resonance imaging D. Digital subtraction angiography E. Computed tomography angiography
C. Gradient echo magnetic resonance imaging Discussion: It is now widely accepted that familial clustering with an autosomal dominant inheritance pattern should be suspected in cases of multiple cavernous malformations. Magnetic resonance gradient-echo sequences should be considered the method of choice for diagnosis of familial cavernous malformations. Although it may exaggerate the size of the lesion that has a surrounding hemosiderin rim, this technique is most sensitive to detect small lesions. Other listed imaging modalities may miss these small lesions.
127
A 68-year-old man presents with progressive neurologic deficit due to a spinal dural arteriovenous fistula fed by the left L4 radicular artery. Treatment for the spinal dural AV fistula requires occlusion of which vascular structure? A. The anterior spinal artery B. The arterial feeder immediately proximal to the fistula C. The draining vein immediately distal to the fistula. D. The L4 radicular artery E. The draining vein in the spinal cord.
C. The draining vein immediately distal to the fistula. Discussion: The correct answer is the draining vein immediately distal to the fistula. The goal of management of spinal dural arteriovenous fistulas is obliteration of the exact site of the fistula, including the early draining vein. This could be accomplished by endovascular transarterial embolization, open surgical clipping, or a combination of both techniques. In most cases, symptomatic improvement is evident following obliteration of the fistula. Obliteration of the proximal arterial pedicle, the radicular artery or the draining vein in the spinal cord are not effective managements for these lesions. Occlusion of the anterior spinal artery would risk paraplegia.
128
A 65 year old female presents with a spontaneous basal ganglia hemorrhage. Her blood pressure is 190/110 mmHg. The patient is not exhibiting signs of intracranial hypertension. How should the blood pressure of the patient be managed? A. Reduce her blood pressure to less than 160/90 mmHg B. Reduce her blood pressure to less than 120/80 mmHg C. Maintain current blood pressure D. Reduce her blood pressure to less than 185/102 mmHg E. Target a mean arterial pressure of 120 mmHg
A. Reduce her blood pressure to less than 160/90 mmHg Discussion: According to the American Heart Association Guidelines for the Management of Spontaneous Intracerebral Hemorrhage, the treatment of elevated blood pressure in patients presenting with spontaneous ICH should follow these recommendations: ● If SBP > 200 mmHg or MPA> 150 mmHg, perform aggressive BP reduction with continuous intravenous infusion and monitor BP every 5 minutes. ● If SBP> 180 mmHg or MAP > 130 mmHg and there is a possibility of elevated ICP, then consider monitoring ICP and reducing BP using intermittent or continuous intravenous medications while maintaining a cerebral perfusion pressure equal or more than 60mm Hg. ● If SBP > 180 mmHg or MAP more than 130 mmHg, and the patient is not exhibiting evidence of elevated ICP then consider modest reduction of BP (target MAP of 110 mmHg or BP 160/90) using intermittent or continuous intravenous medications to control BP and clinically reexamine patients after 15 minutes. The Interact2 (Intensive Blood Pressure Reduction in Acute Cerebral Haemorrhage) Trial randomized 2839 patients who had a spontaneous ICH within 6 hours and who had an elevated systolic blood pressure to receive either intensive treatment (lower systolic blood pressure to target <140mmHG within 1 hour) or standard treatment (lower systolic blood pressure to target <180mmHG). Among the 2794 participants for whom the primary outcome could be determined, 719 of 1382 participants (52.0%) receiving intensive treatment, as compared with 785 of 1412 (55.6%) receiving guideline-recommended treatment, had a primary outcome event (odds ratio with intensive treatment, 0.87; 95% confidence interval [CI], 0.75 to 1.01; P=0.06). Mortality was 11.9% in the group receiving intensive treatment and 12.0% in the group receiving guideline-recommended treatment. In patients with intracerebral hemorrhage, intensive lowering of blood pressure did not result in a significant reduction in the rate of the primary outcome of death or severe disability.
129
What is the most significant risk factor for normal perfusion pressure breakthrough after AVM resection? A. Infratentorial location B. Single feeding artery C. Single draining vein D. Deep venous drainage E. Large size
E. Large size Discussion: Normal perfusion pressure breakthrough can occur after resection of brain arteriovenous malformations and is characterized by post-op swelling or hemorrhage. It is thought to be due to loss of autoregulation in vessels of the surrounding brain parenchyma. Staged embolization and premedication of the patient with propranolol 20 mg PO QID for 3 days before the surgery can decrease the risk of having normal pressure breakthrough post operatively. The larger the size, the higher the risk of normal perfusion pressure to occur. The location of the lesion and the deep venous drainage are not correlated with the risk of Normal perfusion pressure after the resection. Single draining veins and few feeding arteries correlate with a small size and thus predispose to less risk to develop the normal perfusion pressure.
130
According to the International Study of Unruptured Intracranial Aneurysms, what is the 5 year cumulative rupture rate of a 10 mm posterior communicating aneurysm? A. 0% B. 19.5% C. 4.5% D. 14.5% E. 9.5%
D. 14.5% Discussion: According to the International Study of Unruptured Intracranial Aneurysms Investigators, a Posterior Communicating Artery aneurysm measuring between 7-12 mm in its maximum dimension has 14.5% 5-years cumulative rupture risk
131
According to the NASCET trial, what is the best treatment for a symtpmatic carotid artery narrowing of 25%? A. Antiplatelet therapy B. Carotid endarterectomy C. EC/IC bypass D. Carotid artery stenting E. Observation
A. Antiplatelet therapy Discussion: According to NASCET, the best medical management with antiplatelet therapy is the appropriate treatment for symptomatic carotid artery stenosis less than 30%. The same study found that CEA is beneficial for symptomatic disease 70-99% and moderately beneficial for disease 50-69% The NASCET did not investigate the role of carotid artery stenting.
132
What is the most common presenting symptom associated with vain of Galen malformation in a newborn? A. Congestive heart failure B. Seizures C. Headache D. Raised intracranial pressure E. Focal neurological deficit
A. Congestive heart failure Discussion: Vein of Galen malformations are midline arteriovenous fistulae with aneurysmal dilatation of the vein of Galen. The most common presenting symptoms in newborns include congestive cardiac failure (47%), raised intracranial pressure secondary to hydrocephalus (40%), cranial bruit (25%), focal neurological deficit (16%), seizures (11%), and hemorrhage (10%).
133
A 70-year-old female reports that she felt like a curtain dropped over her right eye and made her lose vision. CTA showed 70 % stenosis in her right petrous intracranial carotid artery. Her most recent LDL level is 190 mg/dl and blood pressure 120/75 mmHg. What is the best next step in managing the patient? A. Aspirin, clopidogrel, rosuvastatin B. Percutaneous carotid artery stenting C. Discharge home and re-evaluate if symptoms recur D. Vessel Bypass E. Carotid Endarterectomy
A. Aspirin, clopidogrel, rosuvastatin Discussion: Based on results from the Stenting Versus Aggressive Medical Therapy for Intracranial Arterial Stenosis Trial (SAMMPRIS), patients with severe stenosis (70-99%) and who experienced a recent transient ischemic attack (like our patient) or stroke due to the carotid stenosis are best managed aggressive medical management alone compared to aggressive medical management and percutaneous stenting. Aggressive medical therapy was defined as ASA 325 mg QD, clopidogrel 75 mg QD and correction of other underlying risk factors such as hypercholesterolemia, hypertension, diabetes and smoking. Enrollment in the study stopped after 451 patients underwent randomization because the 30 day rate of stroke or death was 14.7% in those who had stenting and aggressive medical therapy compared to 5.8% to those who received aggressive medical therapy alone. If medical therapy fails, then stenting can be considered for this patient
134
A 51 year old man presents with a severe thunderclap headache, meningismus, photophobia and nausea. His CT scan is shown below and his cerebral angiogram is negative. What is the appropriate treatment option at presentation? A. Surgical exploration B. Hyperdynamic therapy C. Endovascular treatment D. Anticonvulsants E. Observation
E. Observation Discussion: Perimesencephalic nonaneurysmal subarachnoid hemorrhage is associated with low risk of rebleeding and delayed ischemia. Thus measures such as those listed are not indicated. Observation and symtomatic therapy, correction of electrolytes, cardiac monitoring and follow up to rule out hydrocephalus are the mainstay of management.
135
What is a known genetic predisposition to cavernous malformations? A. IDH1 gene B. There is no genetic predisposition C. KRIT1 gene D. VHL gene E. BRAF gene
C. KRIT1 gene Discussion: There is definitely genetic predisposition in cavernous malformations, especially in Hispanics and CCM1. CCM 1 locus is on 7q11. The most likely candidate gene is KRIT1, which encodes a novel Krev-1/rap1a binding protein. CCM2 locus is on 7p13 while CCM3 locus is on 3q25. VHL is the candidate gene for Von Hippel Lindau syndrome. IDH1 and BRAF mutations have important implications in prognosis of patients with gliobastoma and melanoma respectively.
136
What is the most common presentation of a vein of Galen malformation in an adolescent patient? A. Focal neurological deficit B. Hemorrhage C. Heart failure D. Hydrocephalus E. Headache or seizure
E. Headache or seizure Discussion: Older children and adults with vein of Galen malformations usually have low-flow fistulae. These patients usually present with headache and seizures. A small number of patients may also present with developmental delay, focal neurological deficits, proptosis and epistaxis. Subarachnoid hemorrhage and intracerebral hemorrhage can occur in this age group due to rerouting of blood into the pial veins. The most common presenting symptoms in infants include congestive cardiac failure (47%), raised intracranial pressure secondary to hydrocephalus (40%), cranial bruit (25%), focal neurological deficit (16%), seizures (11%), and hemorrhage (10%).
137
A 24 year old pregnant woman in her third trimester presents with severe headache and a non-focal neurological examination. A head CT angiogram is shown. What is the most likely diagnosis? A. Eclampsia B. Dural venous sinus thrombosis C. Aneurysmal subarachnoid hemmorrhage D. Posterior reveresible encephalopathy syndrome E. Meningioma
B. Dural venous sinus thrombosis Discussion: The patient suffers from a dural venous sinus thrombosis. This is a life-threatening emergency that results from clotting in the venous outflow tracts of the brain, namely the dural venous sinuses. When the superior sagittal sinus becomes occluded by clot (thrombosis), venous hypertension develops and a massive hemorrhagic stroke can occur. The thrombosis can be seen as a filling defect (“empty delta sign”) in the superior sagittal sinus on imaging scans. Treatment in this patient requires immediate anti-coagulation. Venous thrombectomy can also be considered in life threatening or refractory cases. There are multiple risk factors to develop venous thrombosis, including pregnancy, hormone replacement therapy, dehydration, malignancy, and inflammatory bowel disease. Recent craniotomy near a venous sinus is also a risk factor. There is no tumor seen on this imaging. Eclampsia is the onset of seizures, typically tonic/clonic, in a woman with pre-eclampsia. Cerebral hemorrhage can also be seen in eclampsia. Posterior reversible encephalopathy syndrome is characterized by headaches and visual loss. It is often caused by malignant hypertension and has a classic MRI appearance of T2 hyperintensities involving the parieto-occiptal lobes bilaterally.
138
What is the maximal total dose of IV rtPA that can be administered for acute ischemic stroke? A. 1.1 mg/kg B. 0.8 mg/kg C. 0.7 mg/kg D. 0.9 mg/kg E. 1 mg/kg
D. 0.9 mg/kg Discussion: According to the NINDS trial, the maximal dose of IV rtPA that can be administered is 90mg. Furthermore, the current guidelines for managing acute ischemic stroke state that intravenous rtPA of 0.9mg/kg is recommended for selected patients who may be treated within 3 hours of onset of ischemic stroke (Class I; Level of Evidence A).
139
Mutation of which of the following genes has been linked to cerebral amyloid angiopathy? A. Tau B. Apoliporprotein E C. Presenilin (PS) D. Endoglin E. KRIT-1
C. Presenilin (PS) Discussion: Cerebral amyloid angiopathy is a form of angiogpahty in which amyloid AB deposits in cortical capillaries. Mutations in the amyloid precursor protein, and presenilin 1 and 2 can result in increased rates of cleaveage of APP into AP amyloid. These findings have been linked to amyloid angiopathy. Mutations in Apolipoprotien E gene leads to familial dysbetalipoproteinemia. Mutations in LDL gene lead to hypercholesterolemia. Mutations in Tau protiens are observed in various numbers of neurodegenerative diseases such as Alzheimer, Pick’s disease, Progressive Supranuclear Palsy etc. Mutations in the KRIT1 gene account for up to 50 percent of all familial cerebral cavernous malformation cases. More than 100 KRIT1 gene mutations have been identified in families with cerebral cavernous malformations
140
When comparing carotid endarterectomy (CEA) to carotid artery stenting (CAS), what is the main difference between the two procedures in terms of stroke and cardiovascular complications? A. Stroke had a greater adverse effect than did myocardial infarction B. CAS was associated with more myocardial infarction C. CEA was associated with more strokes D. The rate of overall complications were higher in CAS than CEA E. CAS was associated with higher incidence of vessel rupture
A. Stroke had a greater adverse effect than did myocardial infarction Discussion: The major results of the CREST trial included: The periprocedural rate of stroke was higher with stenting while the rate of myocardial infarction was higher with endarterectomy. All together, CEA and CAS had similar rates of periprocedural complications. Furthermore, the quality of life analyses among survivors at 1 year indicated that stroke had a greater adverse effect on broad range of health status domains than did the myocardial infarction.
141
Which anatomic feature best predicts risk of intracranial hemorrhage related to dural arteriovenous fistulae (DAVF): A. number of arterial feeders B. presence of sinus drainage C. number of draining veins D. presence of cortical venous drainage E. size of the fistula
D. presence of cortical venous drainage Discussion: The two most common classifications for dural AVFs are the Borden and the Cognard classification shown below Borden Classification Column1 1 Venous drainage directly into venous sinus or meningeal veins 2 Venous drainage into dural venous sinus wirth CVR 3 Venous Drainage directly into subarachnoid veins (CVR) only Cognard Classification I Venous drainage into dural venous sinus with antegrade flow IIa Venous drainage into dural venous sinus with retrograde flow IIb Venous drainage into dural venous sinus with antegrade flow and CVR IIa+b Venous drainage into dural venous sinus with retrograde flow and CVR IV Tyoe III with venous ectasias of the draining subarachnoid venis V Direct drainage into spinal perimedullary veins (CVR indicates cortical veous reflux) Presence of cortical venous drainage is the most important determinant of rupture risk from dural arteriovenous fistulae. The other factors listed have not been convincingly linked to risk of hemorrhage.
142
An 80-year-old female presents with an occipital lobe hemorrhage on CT imaging. She has a prior history of parietal and temporal hemorrhages. What is the most likely diagnosis? A. Coagulopathy B. Venous angioma C. Amyloid angiopathy D. Vasculitis E. Arteriovenous malformation
C. Amyloid angiopathy Discussion: Cerebral amyloid angiopathy should be suspected in elderly patients with recurrent hemorrhages that are lobar in location. Definitive diagnosis is achieved with a brain biopsy that shows deposition of beta amyloid protein appearing as birefringent “apple-green” under polarized light when stained with Congo red. The magnetic resonance imaging can lead to possible or probable diagnosis of cerebral amyloid angiopathy. Arteriovenous malformation hemorrhage would recur in the same anatomical location. Venous angioma is an incidental finding and does not present with hemorrhage. Vasculitis and coagulopathy may lead to recurrent intracranial hemorrhage, however they are often associated with other systemic disorders.
143
A 67 year old man presents to the emergency room with worsening parasthesias of the ipsilateral face and contralateral extremity. The symptoms are provoked with head turning. Where is the most likely site of vascular compression in this patient? A. Posterior inferior cerebellar artery B. Subclavian artery C. Basilar artery D. Carotid Bifurcation E. Dominant vertebral artery
E. Dominant vertebral artery Discussion: The patient has Bow Hunter syndrome and the most likely site of compression is the dominant vertebral artery. Most cases occur at C1-2 with head rotation, but can occur at lower cervical levels. Constriction of the vertebral artery can be caused by mechanical narrowing, bone spurs, tumors, fibrous bands, infection, or trauma. Symptoms are due to vertebrobasilar insufficiency from intermittent vertebral artery occlusion with head rotation. Most patients will not have a complete circle of Willis and will have an isolated posterior circulation. The dominant vertebral artery is usually the pathological side and a stenotic or hypoplastic contralateral vertebral artery is often seen. Symptoms will develop when the head is turned to the contralateral side as this compresses the artery maximally. Dynamic cerebral angiography with head turning will demonstrate flow limitation or occlusion when the head is turned to the contralateral side, however is associated with a slight increase in risk of TIA/stroke compared to a standard cerebral angiogram. Patients should be aware of this when consenting to the study. Angiogram will usually demonstrate an absent or hypoplastic posterior communicating arteries. CT/CTA and MR imaging may be helpful in demonstrating a bony spur or other cause of compression that may be amenable to surgical decompression. Cross section vascular imaging has the benefit of demonstrating relations of the vascular structures and the offending structure. Management options include decompression of the offending structure in the case of the tumors and fibrous bands and bony spurs. Decompression of the bony foramen transversarium has also been used successfully. Soft tissue tethering or fibrous structures along the foramen transversarium or sulcus arteriosus are also viable options. Anticoagulation may be effective but is often not sufficient for complete symptom resolution. Hemilaminectomy at C1 to decompress the vertebral artery is another option to free up the artery. Cervical fusion at C1-2 immobilizes the segment and prevents dynamic compression at the cost of loss of neck rotation. As always; the best option for the patient is determined by the cause, the clincial status of the patient, and the risks involved in the procedure
144
What is the most common location of a cavernous malformation of the central nervous system? A. Brainstem B. Cerebral hemisphere C. Cerebellum D. Optic Pathway E. Spinal cord
B. Cerebral hemisphere Discussion: The most common location of cavernoma is supratentorial (48-86%). Brainstem cavernoma occur in 4-35% and basal ganglia cavernomas in 5-10%. Spinal cavernomas account for less than 5% of all cavernous malformations. Optic pathway cavernomas are exceedingly rare (<1%), and typically involve the chiasm/hypothalamus (<1%).
145
A 54-year-old man presents with acute onset of right neck pain radiating to his jaw. He denies any other symptoms. His clinical findings are demonstrated on the photograph below. Which of the following would be the most appropriate imaging study to order? A. CT angiogram of the neck B. MRI of the cervical spine C. MRI of the brain and orbits D. CT scan of the chest E. CT angiogram of the head
A. CT angiogram of the neck Discussion: The correct answer is CT angiogram of the neck. This patient has a right Horner syndrome with mild upper lid ptosis and miosis. The presence of a Horner syndrome associated with neck pain should raise the suspicion of carotid artery dissection and urgent investigation is required. CT angiogram of the neck is the study of choice in most institutions. Alternatively, a MR angiogram of the neck or axial MRI of the neck without contrast and with fat suppression represents a very good alternative for investigation of potential carotid artery dissection. While a chest CT scan, cervical spine MRI, and MRI of the brain and orbits can certainly be considered for the investigation of Horner syndrome, none of these imaging studies should come first when a carotid dissection is suspected. A thorough history alone may help determine the etiology of Horner syndrome, for example if there has been previous accidental or surgical trauma to the chest, neck, or upper spine. No further work-up is necessary if such history is obtained, although it is helpful to document that the Horner syndrome is temporally related to the surgery or trauma by checking old photographs. Associated signs and symptoms might help localize the lesion. In a central Horner syndrome there will be associated neurological findings. The presence of ataxia, nystagmus, and hemisensory deficit, for example, would strongly suggest a medullary lesion and an MRI of the brain would be recommended. An acute Horner syndrome associated with ipsilateral facial or neck pain requires urgent imaging of the neck to exclude a carotid dissection or thrombosis. Trigeminal autonomic syndromes such as cluster headache should remain diagnoses of exclusion since carotid dissections can present in a similar fashion. Arm pain, weakness and numbness would suggest a lesion near the lung apex, brachial plexus or cervical spine. The presence of an ipsilateral sixth, third, or fourth nerve palsy or trigeminal dysfunction would suggest a lesion in the cavernous sinus and should be further evaluated with MRI. In a patient with isolated Horner syndrome, a chest and neck study should be performed. An adequate neck protocol should go as far up as the skull base. CT/CTA of the neck is a good choice since CT offers excellent resolution of the soft tissues of the neck and CTA provides good views of the carotid artery lumen. Alternatively, MRI/MRA of the neck (including the skull base) along with a chest CT would also provide excellent evaluation of an isolated Horner syndrome. A mass lesion such as neuroblastoma is the main concern in the presence of Horner syndrome in a child of any age without a surgical history. Urine catecholamine levels alone cannot rule in or out neuroblastoma. Emphasis should be placed on a thorough physical examination and imaging studies of the brain, neck, and chest. MRI is the modality of choice in the pediatric population.
146
Occlusion of which artery causes the stroke shown in the figure? A. Recurrent artery of Heubner B. Medial lenticulostriate perforator C. Subcallosal artery D. Orbitofrontal Artery E. Artery of Percheron
C. Subcallosal artery Discussion: Occlusion of the subcallosal artery after anterior communicating artery aneurysm clipping may cause a forniceal stroke leading to severe anterograde and retrograde amnesia. Compromise of the recurrent artery of Heubner causes a stroke of the caudate and anterior limb of the internal capsule. Medial lenticulostriate perforator occlusions may lead to a stroke in the caudate, anterior limb and/or genu of the internal capsule. Occlusion of the artery of Percheron causes a thalamic stroke, and occlusion of the orbitofrontal artery causes to a subfrontal stroke.
147
What factor increases the risk of future hemorrhage from an AVM? A. AVM location B. Spetzler Martin Grade C. Female gender D. Older age E. Previous hemorrhage
E. Previous hemorrhage Discussion: The literature strongly suggests that the risk factor “prior hemorrhage” has been associated with the greatest increase in subsequent hemorrhage risk of the listed factors above. . Older age, AVM location, Spetzler Martin Grade have not been convincingly linked to AVM rupture risk. The Spetzler Martin Grade relies on a number of features (size, eloquence of adjacent brain and venous drainage) of intracranial AVMs to correlate grade with microsurgical outcomes.
148
In the acute workup of clinically suspected subarachnoid hemorrhage what would be the most appropriate next step in evaluation following a negative head CT? A. MRA circle of Willis B. Lumbar Puncture for CSF analysis C. MRI brain with GRE sequence D. CT angiogram E. CT with contrast
B. Lumbar Puncture for CSF analysis Discussion: According to the latest SAH guidelines, a negative CT after a suspected history of SAH should be followed by a lumbar puncture (Class I recommendation Level B). The sensitivity of CT in the first 3 days after a SAH remains very high (close to 100%), after which it decreases moderately during the next few days. After 5 to 7 days, the rate of negative CT increases sharply, and lumbar puncture is often required to show xanthochromia.
149
A 45 year old man presents with sudden onset of vomiting, dysarthria and ataxia and a CT scan demonstrating a cerebellar infarction. Thirteen hours after onset of symptoms, he develops abducens nerve palsy and depressed mental status. What is the most appropriate definitive management of this patient? A. Intravenous tPA B. Barbiturate coma C. Suboccipital craniectomy D. High dose Dexamethasone E. Hypertonic saline
C. Suboccipital craniectomy Discussion: Surgery for cerebellar infarction should be performed if the patient does not respond to aggressive medical management or if the patient develops any of the following symptoms: abducens nerve palsy, loss of ipsilateral gaze, peripheral facial nerve paresis, hemiparesis, lethargy, small but reactive pupils, coma, flaccidity or ataxic respiration. The surgery of choice is suboccipital decompressive craniectomy. The clinical picture of the patient shows that he is deteriorating. Thus suboccipital craniectomy with dural expansion should be performed in all patients who continue to deteriorate. Ventriculostomy to relieve obstructive hydrocephalus after cerebellar infarct should be accompanied by decompressive suboccipital craniectomy to avoid deterioration and upward cerebellar deviation. There is no role for dexamethasone therapy for ischemic stroke.
150
During surgical resection of a brainstem cavernous malformation, a developmental venous anomaly (DVA) was detected. Which of the following is an appropriate step to take during surgery? A. Perform an intraoperative angiogram B. Coagulate the associating DVA after removing the cavernous malformation C. Perform indocyanine green (ICG) video angiography D. Apply a clip to the DVA prior to removal of cavernous malformation E. Preserve the associated DVA
E. Preserve the associated DVA Discussion: Developmental venous anomalies (DVA) drain normal brain and need to be preserved during surgery to prevent venous infarction. Surgical resection of DVAs associated with brainstem cavernous malformations has been reported to result in hemorrhagic infarction of the brainstem. DVAs provide venous drainage of normal brain tissue. Clipping and coagulation of DVAs may suddenly interrupt the venous drainage of normal parenchyma and precipitate venous infarction with edema and the potential for devastating consequences.